Что делается в первую очередь деление или умножение: Что сперва умножение или деление — РОСТОВСКИЙ ЦЕНТР ПОМОЩИ ДЕТЯМ № 7

Содержание

Действия какой ступени выполняются в первую очередь?

Ответьте на вопрос: —

Какой порядок действий при решении длинного выражения?

Порядок выполнения действий при решении выражения.

В математике последовательность выполнения действий разделена на две ступени:
к первой ступени относятся действия — сложение и вычитание, ко второй ступени — умножение и деление.
При нахождении значения выражения в первую очередь выполняются действия заключённые в скобки (если имеются), далее выполняются действия второй ступени и в последнюю очередь действия первой ступени.
Порядок действий обозначается слева направо.

Пример:

1. Слева направо обозначим действия в скобках.

2. Вернёмся к началу примера и снова продолжим слева направо обозначать теперь действия второй ступени.
Действия второй ступени — умножение и деление.

3. Вновь вернёмся к началу примера и снова продолжим слева направо обозначать теперь действия первой ступени.
Действия первой ступени — сложение и вычитание.

Всего 10 действий. Выполняем их по проставленному порядку.

Выполним действия в скобках:

18 + 8 : (27 — 25) — 2 · 8 + 4 · (6 + 4) + 16 : 8

  1. 27 — 25 = 2
  2. 6 + 4 = 10

    После выполнения действий в скобках выражение стало выглядеть так:

    18 + 8 : 2 — 2 · 8 + 4 · 10 + 16 : 8

    Теперь выполним все действия второй ступени — умножение и деление:

    18 + 8 : 22 · 8 + 4 · 10 + 16 : 8

  3. 8 : 2 = 4
  4. 2 · 8 = 16
  5. 4 · 10 = 40
  6. 16 : 8 = 2

    После выполнения действий второй ступени выражение стало выглядеть так:

    18 + 4 — 16 + 40 + 2

    Теперь выполним все действия первой ступени — сложение и вычитание:

    18 + 4 — 16 + 40 + 2

  7. 18 + 4 = 22
  8. 22 — 16 = 6
  9. 6 + 40 = 46
  10. 46 + 2 = 48

    18 + 8 : (27 — 25) — 2 · 8 + 4 · (6 + 4) + 16 : 8 = 48

Коротко:

Известные и великие математики

ученые средневековья и современности, и их вклад в мировую науку

Пафнутий Чебышёв

Русский математик и механик
Дата рождения: 16 мая 1821
Место рождения: Акатово, Боровский уезд, Калужская губерния, Российская империя
Дата смерти: 8 декабря 1894 (73 года)

Биография

Первоначальное воспитание и образование получил дома: грамоте его обучила мать Аграфена Ивановна. Арифметике, французскому языку и музыке обучала двоюродная сестра Авдотья Квинтилиановна Сухарёва. Одним из детских увлечений будущего учёного было изучение механизмов игрушек и автоматов, которые сам придумывал и изготовлял их.

В 1832 году семья переехала в Москву. В Москве с Пафнутием математикой и физикой занимался П. Н. Погорельский — один из лучших учителей Москвы, у которого в том числе учился, в пансионе Вейденгаммера, и И. С. Тургенев. Латынь Пафнутию Чебышёву преподавал в то время студент-медик, а в будущем главный врач Шереметевской больницы А. Т. Тарасенков.

Летом 1837 года Чебышёв поступил в Императорский Московский университет на вторе физико- математическе отделение философского факультета и начал изучение математики . Существенное влияние на формирование круга научных интересов молодого Чебышёва оказал его учитель — профессор прикладной математики и механики Московского университета Николай Дмитриевич Брашман. В 1841 году Пафнутий Чебышёв его окончил.

В 1846 году он успешно защитил магистерскую диссертацию «Опыт элементарного анализа теории вероятностей». В 1847 году Чебышёв был утверждён в звании адъюнкт-профессора Петербургского университета. Чтобы получить право чтения лекций в университете, он защитил ещё одну диссертацию — на тему «Об интегрировании с помощью логарифмов», после чего читал лекции по высшей алгебре, теории чисел, геометрии, теории эллиптических функций и практической механике.

В 1846 году он успешно защитил магистерскую диссертацию «Опыт элементарного анализа теории вероятностей». В 1847 году Чебышёв был утверждён в звании адъюнкт-профессора Петербургского университета. Чтобы получить право чтения лекций в университете, он защитил ещё одну диссертацию — на тему «Об интегрировании с помощью логарифмов», после чего читал лекции по высшей алгебре, теории чисел, геометрии, теории эллиптических функций и практической механике.

В 1849 году Чебышёв защитил в Петербургском университете докторскую диссертацию «Теория сравнений», после чего в 1850 году он стал профессором Петербургского университета; данную должность он занимал до 1882 года. Работая в Петербургском университете, Чебышёв близко сошёлся с профессором прикладной математики О. И. Сомовым, который тоже был учеником Н. Д. Брашмана, и эти отношения переросли в глубокую дружбу. В семейном плане Чебышёв был одинок, и это обстоятельство также способствовало его сближению с большой семьёй Сомова.

Интерес к механизмам сохранялся у Чебышёва и в зрелые годы. В 1852 году Чебышёв совершил научную командировку в Великобританию, Францию и Бельгию, в ходе которой он ознакомился с практикой зарубежного машиностроения, с музейными коллекциями машин и механизмов, с работой заводов и фабрик, а также встречался с крупнейшими математиками и механиками: О. Коши, Ж. Лиувиллем, Ж.-А. Серре, Л. Фуко, Ш. Эрмитом, Дж. Сильвестром, А. Кэли, Т. Грегори. После этого он некоторое время преподавал практическую механику в Петербургском университете и Александровском лицее.

В 1853 году академики П. Н. Фусс, В. Я. Струве, Б. С. Якоби, В. Я. Буняковский представили Чебышёва к избранию в адъюнкты Петербургской академии наук, особо отметив важность его работ в области практической механики. В том же году он был избран в адъюнкты, а в 1856 году стал экстраординарным академиком.

В 1858 году в связи с его работами по теории шарнирных параллелограммов и теории приближения функций академики В. Я. Буняковский, М. В. Остроградский, Э. Х. Ленц, Б. С. Якоби, А. Я. Купфер, О. В. Струве подписали представление к избранию Чебышёва ординарным академиком. И 1859 году Чебышёв избран ординарным академиком. Стал почётным членом Московского университета.

С 22 февраля 1860 года — ординарный профессор.

С 10 июля 1863 года — член Учёного комитета Министерства народного просвещения.

С 30 августа 1863 года — действительный статский советник.

Чем знаменит:

  • В 1840/1841 учебном году, участвуя в студенческом конкурсе Императорского Московского университета, Пафнутий Чебышёв получил серебряную медаль за работу по нахождению корней уравнения n-й степени которую написал ещё в 1838 году и сделаную на основе алгоритма Ньютона
  • Работы по теории вероятностей — изъяв из неё расплывчатые формулировки и неправомерные утверждения и превратив её в строгую математическую дисциплину
  • Работы по теории чисел
  • Работы по математическому анализу
  • Работы по прикладной математике и механике
  • Работы по «стопоходящей машины»
  • Создатель автоматического арифмометра
  • оздатель модели инвалидной коляски
  • оздатель
  • Работы по

Если заметили ошибку, выделите фрагмент текста и нажмите Ctrl+Enter

Что делается первым действием. Порядок выполнения действий в выражениях без скобок и со скобками

Мы рассмотрим в этой статье три варианта примеров:

1. Примеры со скобками (действия сложения и вычитания)

2. Примеры со скобками (сложение, вычитание, умножение, деление)

3. Примеры, в которых много действий

1 Примеры со скобками (действия сложения и вычитания)

Рассмотрим три примера. В каждом из них порядок действий обозначен цифрами красного цвета:

Мы видим, что порядок действий в каждом примере будет разный, хотя числа и знаки одинаковые. Это происходит потому, что во втором и третьем примере есть скобки.

*Это правило для примеров без умножения и деления. Правила для примеров со скобками, включающих действия умножения и деления мы рассмотрим во второй части этой статьи.

Чтобы не запутаться в примере со скобками, можно превратить его в обычный пример, без скобок. Для этого результат, полученный в скобках, записываем над скобками, далее переписываем весь пример, записывая вместо скобок этот результат, и далее выполняем все действия по порядку, слева направо:

В несложных примерах можно все эти операции производить в уме. Главное — сначала выполнить действие в скобках и запомнить результат, а затем считать по порядку, слева направо.

А теперь — тренажеры!

1) Примеры со скобками в пределах до 20. Онлайн тренажер.

2) Примеры со скобками в пределах до 100. Онлайн тренажер.

3) Примеры со скобками. Тренажер №2

4) Вставь пропущенное число — примеры со скобками. Тренажер

2 Примеры со скобками (сложение, вычитание, умножение, деление)

Теперь рассмотрим примеры, в которых кроме сложения и вычитания есть умножение и деление.

Сначала рассмотрим примеры без скобок:

Есть одна хитрость, как не запутаться при решении примеров на порядок действий. Если нет скобок, то выполняем действия умножения и деления, далее переписываем пример, записывая вместо этих действий полученные результаты. Затем выполняем сложение и вычитание по порядку:

Если в примере есть скобки, то сначала нужно избавиться от скобок: переписать пример, записывая вместо скобок полученный в них результат. Затем нужно выделить мысленно части примера, разделенные знаками «+» и «-«, и посчитать каждую часть отдельно. Затем выполнить сложение и вычитание по порядку:

3 Примеры, в которых много действий

Если в примере много действий, то удобнее будет не расставлять порядок действий во всем примере, а выделить блоки, и решить каждый блок отдельно. Для этого находим свободные знаки «+» и «–» (свободные — значит не в скобках, на рисунке показаны стрелочками).

Эти знаки и будут делить наш пример на блоки:

Выполняя действия в каждом блоке не забываем про порядок действий, приведенный выше в статье. Решив каждый блок, выполняем действия сложения и вычитания по порядку.

А теперь закрепляем решение примеров на порядок действий на тренажерах!

Если у вас не открываются игры или тренажёры, читайте . В пятом веке до нашей эры древнегреческий философ Зенон Элейский сформулировал свои знаменитые апории, самой известной из которых является апория «Ахиллес и черепаха». Вот как она звучит:

Допустим, Ахиллес бежит в десять раз быстрее, чем черепаха, и находится позади неё на расстоянии в тысячу шагов. За то время, за которое Ахиллес пробежит это расстояние, черепаха в ту же сторону проползёт сто шагов. Когда Ахиллес пробежит сто шагов, черепаха проползёт ещё десять шагов, и так далее. Процесс будет продолжаться до бесконечности, Ахиллес так никогда и не догонит черепаху.

Это рассуждение стало логическим шоком для всех последующих поколений. Аристотель, Диоген, Кант, Гегель, Гильберт… Все они так или иначе рассматривали апории Зенона. Шок оказался настолько сильным, что «… дискуссии продолжаются и в настоящее время, прийти к общему мнению о сущности парадоксов научному сообществу пока не удалось… к исследованию вопроса привлекались математический анализ, теория множеств, новые физические и философские подходы; ни один из них не стал общепризнанным решением вопроса… » [Википедия, » Апории Зенона «]. Все понимают, что их дурят, но никто не понимает, в чем заключается обман.

С точки зрения математики, Зенон в своей апории наглядно продемонстрировал переход от величины к . Этот переход подразумевает применение вместо постоянных. Насколько я понимаю, математический аппарат применения переменных единиц измерения либо ещё не разработан, либо его не применяли к апории Зенона. Применение же нашей обычной логики приводит нас в ловушку. Мы, по инерции мышления, применяем постоянные единицы измерения времени к обратной величине. С физической точки зрения это выглядит, как замедление времени до его полной остановки в момент, когда Ахиллес поравняется с черепахой. Если время останавливается, Ахиллес уже не может перегнать черепаху.

Если перевернуть привычную нам логику, всё становится на свои места. Ахиллес бежит с постоянной скоростью. Каждый последующий отрезок его пути в десять раз короче предыдущего. Соответственно, и время, затрачиваемое на его преодоление, в десять раз меньше предыдущего. Если применять понятие «бесконечность» в этой ситуации, то правильно будет говорить «Ахиллес бесконечно быстро догонит черепаху».

Как избежать этой логической ловушки? Оставаться в постоянных единицах измерения времени и не переходить к обратным величинам. На языке Зенона это выглядит так:

За то время, за которое Ахиллес пробежит тысячу шагов, черепаха в ту же сторону проползёт сто шагов. За следующий интервал времени, равный первому, Ахиллес пробежит ещё тысячу шагов, а черепаха проползет сто шагов. Теперь Ахиллес на восемьсот шагов опережает черепаху.

Этот подход адекватно описывает реальность без всяких логических парадоксов. Но это не полное решение проблемы. На Зеноновскую апорию «Ахиллес и черепаха» очень похоже утверждение Эйнштейна о непреодолимости скорости света. Эту проблему нам ещё предстоит изучить, переосмыслить и решить. И решение нужно искать не в бесконечно больших числах, а в единицах измерения.

Другая интересная апория Зенона повествует о летящей стреле:

Летящая стрела неподвижна, так как в каждый момент времени она покоится, а поскольку она покоится в каждый момент времени, то она покоится всегда.

В этой апории логический парадокс преодолевается очень просто — достаточно уточнить, что в каждый момент времени летящая стрела покоится в разных точках пространства, что, собственно, и является движением. Здесь нужно отметить другой момент. По одной фотографии автомобиля на дороге невозможно определить ни факт его движения, ни расстояние до него. Для определения факта движения автомобиля нужны две фотографии, сделанные из одной точки в разные моменты времени, но по ним нельзя определить расстояние. Для определения расстояния до автомобиля нужны две фотографии, сделанные из разных точек пространства в один момент времени, но по ним нельзя определить факт движения (естественно, ещё нужны дополнительные данные для расчетов, тригонометрия вам в помощь). На что я хочу обратить особое внимание, так это на то, что две точки во времени и две точки в пространстве — это разные вещи, которые не стоит путать, ведь они предоставляют разные возможности для исследования.

среда, 4 июля 2018 г.

Очень хорошо различия между множеством и мультимножеством описаны в Википедии . Смотрим.

Как видите, «во множестве не может быть двух идентичных элементов», но если идентичные элементы во множестве есть, такое множество называется «мультимножество». Подобную логику абсурда разумным существам не понять никогда. Это уровень говорящих попугаев и дрессированных обезьян, у которых разум отсутствует от слова «совсем». Математики выступают в роли обычных дрессировщиков, проповедуя нам свои абсурдные идеи.

Когда-то инженеры, построившие мост, во время испытаний моста находились в лодке под мостом. Если мост обрушивался, бездарный инженер погибал под обломками своего творения. Если мост выдерживал нагрузку, талантливый инженер строил другие мосты.

Как бы математики не прятались за фразой «чур, я в домике», точнее «математика изучает абстрактные понятия», есть одна пуповина, которая неразрывно связывает их с реальностью. Этой пуповиной являются деньги. Применим математическую теорию множеств к самим математикам.

Мы очень хорошо учили математику и сейчас сидим в кассе, выдаем зарплату. Вот приходит к нам математик за своими деньгами. Отсчитываем ему всю сумму и раскладываем у себя на столе на разные стопки, в которые складываем купюры одного достоинства. Затем берем с каждой стопки по одной купюре и вручаем математику его «математическое множество зарплаты». Поясняем математику, что остальные купюры он получит только тогда, когда докажет, что множество без одинаковых элементов не равно множеству с одинаковыми элементами. Вот здесь начнется самое интересное.

В первую очередь, сработает логика депутатов: «к другим это применять можно, ко мне — низьзя!». Дальше начнутся уверения нас в том, что на купюрах одинакового достоинства имеются разные номера купюр, а значит их нельзя считать одинаковыми элементами. Хорошо, отсчитываем зарплату монетами — на монетах нет номеров. Здесь математик начнет судорожно вспоминать физику: на разных монетах имеется разное количество грязи, кристаллическая структура и расположение атомов у каждой монеты уникально…

А теперь у меня самый интересный вопрос: где проходит та грань, за которой элементы мультимножества превращаются в элементы множества и наоборот? Такой грани не существует — всё решают шаманы, наука здесь и близко не валялась.

Вот смотрите. Мы отбираем футбольные стадионы с одинаковой площадью поля. Площадь полей одинакова — значит у нас получилось мультимножество. Но если рассматривать названия этих же стадионов — у нас получается множество, ведь названия разные. Как видите, один и тот же набор элементов одновременно является и множеством, и мультимножеством. Как правильно? А вот здесь математик-шаман-шуллер достает из рукава козырный туз и начинает нам рассказывать либо о множестве, либо о мультимножестве. В любом случае он убедит нас в своей правоте.

Чтобы понять, как современные шаманы оперируют теорией множеств, привязывая её к реальности, достаточно ответить на один вопрос: чем элементы одного множества отличаются от элементов другого множества? Я вам покажу, без всяких «мыслимое как не единое целое» или «не мыслимое как единое целое».

воскресенье, 18 марта 2018 г.

Сумма цифр числа — это пляска шаманов с бубном, которая к математике никакого отношения не имеет. Да, на уроках математики нас учат находить сумму цифр числа и пользоваться нею, но на то они и шаманы, чтобы обучать потомков своим навыкам и премудростям, иначе шаманы просто вымрут.

Вам нужны доказательства? Откройте Википедию и попробуйте найти страницу «Сумма цифр числа». Её не существует. Нет в математике формулы, по которой можно найти сумму цифр любого числа. Ведь цифры — это графические символы, при помощи которых мы записываем числа и на языке математики задача звучит так: «Найти сумму графических символов, изображающих любое число». Математики эту задачу решить не могут, а вот шаманы — элементарно.

Давайте разберемся, что и как мы делаем для того, чтобы найти сумму цифр заданного числа. И так, пусть у нас есть число 12345. Что нужно сделать для того, чтобы найти сумму цифр этого числа? Рассмотрим все шаги по порядку.

1. Записываем число на бумажке. Что же мы сделали? Мы преобразовали число в графический символ числа. Это не математическое действие.

2. Разрезаем одну полученную картинку на несколько картинок, содержащих отдельные цифры. Разрезание картинки — это не математическое действие.

3. Преобразовываем отдельные графические символы в числа. Это не математическое действие.

4. Складываем полученные числа. Вот это уже математика.

Сумма цифр числа 12345 равна 15. Вот такие вот «курсы кройки и шитья» от шаманов применяют математики. Но это ещё не всё.

С точки зрения математики не имеет значения, в какой системе счисления мы записываем число. Так вот, в разных системах счисления сумма цифр одного и того же числа будет разной. В математике система счисления указывается в виде нижнего индекса справа от числа. С большим числом 12345 я не хочу голову морочить, рассмотрим число 26 из статьи про . Запишем это число в двоичной, восьмеричной, десятичной и шестнадцатеричной системах счисления. Мы не будем рассматривать каждый шаг под микроскопом, это мы уже сделали. Посмотрим на результат.

Как видите, в разных системах счисления сумма цифр одного и того же числа получается разной. Подобный результат к математике никакого отношения не имеет. Это всё равно, что при определении площади прямоугольника в метрах и сантиметрах вы получали бы совершенно разные результаты.

Ноль во всех системах счисления выглядит одинаково и суммы цифр не имеет. Это ещё один аргумент в пользу того, что . Вопрос к математикам: как в математике обозначается то, что не является числом? Что, для математиков ничего, кроме чисел, не существует? Для шаманов я могу такое допустить, но для ученых — нет. Реальность состоит не только из чисел.

Полученный результат следует рассматривать как доказательство того, что системы счисления являются единицами измерения чисел. Ведь мы не можем сравнивать числа с разными единицами измерения. Если одни и те же действия с разными единицами измерения одной и той же величины приводят к разным результатам после их сравнения, значит это не имеет ничего общего с математикой.

Что же такое настоящая математика? Это когда результат математического действия не зависит от величины числа, применяемой единицы измерения и от того, кто это действие выполняет.

Открывает дверь и говорит:

Ой! А это разве не женский туалет?
— Девушка! Это лаборатория по изучению индефильной святости душ при вознесении на небеса! Нимб сверху и стрелочка вверх. Какой еще туалет?

Женский… Нимб сверху и стрелочка вниз — это мужской.

Если у вас перед глазами несколько раз в день мелькает вот такое вот произведение дизайнерского искусства,

Тогда не удивительно, что в своем автомобиле вы вдруг обнаруживаете странный значок:

Лично я делаю над собой усилие, чтобы в какающем человеке (одна картинка), увидеть минус четыре градуса (композиция из нескольких картинок: знак минус, цифра четыре, обозначение градусов). И я не считаю эту девушку дурой, не знающей физику. Просто у неё дугой стереотип восприятия графических образов. И математики нас этому постоянно учат. Вот пример.

1А — это не «минус четыре градуса» или «один а». Это «какающий человек» или число «двадцать шесть» в шестнадцатеричной системе счисления. Те люди, которые постоянно работают в этой системе счисления, автоматически воспринимают цифру и букву как один графический символ.

Начальная школа подходит к концу, скоро ребёнок шагнёт в углубленный мир математики. Но уже в этот период школьник сталкивается с трудностями науки. Выполняя простое задание, ребёнок путается, теряется, что в результате приводит к отрицательной отметке за выполненную работу. Чтобы избежать подобных неприятностей, нужно при решении примеров, уметь ориентироваться в порядке, по которому нужно решать пример. Не верно распределив действия, ребёнок не правильно выполняет задание. В статье раскрываются основные правила решения примеров, содержащих в себе весь спектр математических вычислений, включая скобки. Порядок действий в математике 4 класс правила и примеры.

Перед выполнением задания попросите своё чадо пронумеровать действия, которые он собирается выполнить. Если возникли затруднения – помогите.

Некоторые правила, которые необходимо соблюдать при решении примеров без скобок:

Если в задании необходимо выполнить ряд действий, нужно сначала выполнить деление или умножение, затем . Все действия выполняются по ходу письма. В противном случае, результат решения будет не верным.

Если в примере требуется выполнить , выполняем по порядку, слева направо.

27-5+15=37 (при решении примера руководствуемся правилом. Сначала выполняем вычитание, затем – сложение).

Научите ребёнка всегда планировать и нумеровать выполняемые действия.

Ответы на каждое решённое действие записываются над примером. Так ребёнку гораздо легче будет ориентироваться в действиях.

Рассмотрим ещё один вариант, где необходимо распределить действия по порядку:

Как видим, при решении соблюдено правило, сначала ищем произведение, после — разность.

Это простые примеры, при решении которых, необходима внимательность. Многие дети впадают в ступор при виде задания, в котором присутствует не только умножение и деление, но и скобки. У школьника, не знающего порядок выполнения действий, возникают вопросы, которые мешают выполнить задание.

Как говорилось в правиле, сначала найдём произведение или частное, а потом всё остальное. Но тут же есть скобки! Как поступить в этом случае?

Решение примеров со скобками

Разберём конкретный пример:

  • При выполнении данного задания, сначала найдём значение выражения, заключённого в скобки.
  • Начать следует с умножения, далее – сложение.
  • После того, как выражение в скобках решено, приступаем к действиям вне их.
  • По правилам порядка действий, следующим шагом будет умножение.
  • Завершающим этапом станет .

Как видим на наглядном примере, все действия пронумерованы. Для закрепления темы предложите ребёнку решить самостоятельно несколько примеров:

Порядок, по которому следует вычислять значение выражения уже расставлен. Ребёнку останется только выполнить непосредственно решение.

Усложним задачу. Пусть ребёнок найдёт значение выражений самостоятельно.

7*3-5*4+(20-19) 14+2*3-(13-9)
17+2*5+(28-2) 5*3+15-(2-1*2)
24-3*2-(56-4*3) 14+12-3*(21-7)

Приучите ребёнка решать все задания в черновом варианте. В таком случае, у школьника будет возможность исправить не верное решение или помарки. В рабочей тетради исправления не допустимы. Выполняя самостоятельно задания, дети видят свои ошибки.

Родители, в свою очередь, должны обратить внимание на ошибки, помочь ребёнку разобраться и исправить их. Не стоит нагружать мозг школьника большими объёмами заданий. Такими действиями вы отобьёте стремление ребёнка к знаниям. Во всём должно быть чувство меры.

Делайте перерыв. Ребёнок должен отвлекаться и отдыхать от занятий. Главное помнить, что не все обладают математическим складом ума. Может из вашего ребёнка вырастет знаменитый философ.

На данном уроке подробно рассмотрен порядок выполнения арифметических действий в выражениях без скобок и со скобками. Учащимся предоставляется возможность в ходе выполнения заданий определить, зависит ли значение выражений от порядка выполнения арифметических действий, узнать отличается ли порядок арифметических действий в выражениях без скобок и со скобками, потренироваться в применении изученного правила, найти и исправить ошибки, допущенные при определении порядка действий.

В жизни мы постоянно выполняем какие-либо действия: гуляем, учимся, читаем, пишем, считаем, улыбаемся, ссоримся и миримся. Эти действия мы выполняем в разном порядке. Иногда их можно поменять местами, а иногда нет. Например, собираясь утром в школу, можно сначала сделать зарядку, затем заправить постель, а можно наоборот. Но нельзя сначала уйти в школу, а потом надеть одежду.

А в математике обязательно ли выполнять арифметические действия в определенном порядке?

Давайте проверим

Сравним выражения:
8-3+4 и 8-3+4

Видим, что оба выражения совершенно одинаковы.

Выполним действия в одном выражения слева направо, а в другом справа налево. Числами можно проставить порядок выполнения действий (рис. 1).

Рис. 1. Порядок действий

В первом выражении мы сначала выполним действие вычитания, а затем к результату прибавим число 4.

Во втором выражении сначала найдем значение суммы, а потом из 8 вычтем полученный результат 7.

Видим, что значения выражений получаются разные.

Сделаем вывод: порядок выполнения арифметических действий менять нельзя .

Узнаем правило выполнения арифметических действий в выражениях без скобок.

Если в выражение без скобок входят только сложение и вычитание или только умножение и деление, то действия выполняют в том порядке, в каком они написаны.

Потренируемся.

Рассмотрим выражение

В этом выражении имеются только действия сложения и вычитания. Эти действия называют действиями первой ступени .

Выполняем действия слева направо по порядку (рис. 2).

Рис. 2. Порядок действий

Рассмотрим второе выражение

В этом выражении имеются только действия умножения и деления — это действия второй ступени.

Выполняем действия слева направо по порядку (рис. 3).

Рис. 3. Порядок действий

В каком порядке выполняются арифметические действия, если в выражении имеются не только действия сложения и вычитания, но и умножения и деления?

Если в выражение без скобок входят не только действия сложения и вычитания, но и умножения и деления, или оба этих действия, то сначала выполняют по порядку (слева направо) умножение и деление, а затем сложение и вычитание.

Рассмотрим выражение.

Рассуждаем так. В этом выражении имеются действия сложения и вычитания, умножения и деления. Действуем по правилу. Сначала выполняем по порядку (слева направо) умножение и деление, а затем сложение и вычитание. Расставим порядок действий.

Вычислим значение выражения.

18:2-2*3+12:3=9-6+4=3+4=7

В каком порядке выполняются арифметические действия, если в выражении имеются скобки?

Если в выражении имеются скобки, то сначала вычисляют значение выражений в скобках.

Рассмотрим выражение.

30 + 6 * (13 — 9)

Мы видим, что в этом выражении имеется действие в скобках, значит, это действие выполним первым, затем по порядку умножение и сложение. Расставим порядок действий.

30 + 6 * (13 — 9)

Вычислим значение выражения.

30+6*(13-9)=30+6*4=30+24=54

Как нужно рассуждать, чтобы правильно установить порядок арифметических действий в числовом выражении?

Прежде чем приступить к вычислениям, надо рассмотреть выражение (выяснить, есть ли в нём скобки, какие действия в нём имеются) и только после этого выполнять действия в следующем порядке:

1. действия, записанные в скобках;

2. умножение и деление;

3. сложение и вычитание.

Схема поможет запомнить это несложное правило (рис. 4).

Рис. 4. Порядок действий

Потренируемся.

Рассмотрим выражения, установим порядок действий и выполним вычисления.

43 — (20 — 7) +15

32 + 9 * (19 — 16)

Будем действовать по правилу. В выражении 43 — (20 — 7) +15 имеются действия в скобках, а также действия сложения и вычитания. Установим порядок действий. Первым действием выполним действие в скобках, а затем по порядку слева направо вычитание и сложение.

43 — (20 — 7) +15 =43 — 13 +15 = 30 + 15 = 45

В выражении 32 + 9 * (19 — 16) имеются действия в скобках, а также действия умножения и сложения. По правилу первым выполним действие в скобках, затем умножение (число 9 умножаем на результат, полученный при вычитании) и сложение.

32 + 9 * (19 — 16) =32 + 9 * 3 = 32 + 27 = 59

В выражении 2*9-18:3 отсутствуют скобки, зато имеются действия умножения, деления и вычитания. Действуем по правилу. Сначала выполним слева направо умножение и деление, а затем от результата, полученного при умножении, вычтем результат, полученный при делении. То есть первое действие — умножение, второе — деление, третье — вычитание.

2*9-18:3=18-6=12

Узнаем, правильно ли определен порядок действий в следующих выражениях.

37 + 9 — 6: 2 * 3 =

18: (11 — 5) + 47=

7 * 3 — (16 + 4)=

Рассуждаем так.

37 + 9 — 6: 2 * 3 =

В этом выражении скобки отсутствуют, значит, сначала выполняем слева направо умножение или деление, затем сложение или вычитание. В данном выражении первое действие — деление, второе — умножение. Третье действие должно быть сложение, четвертое — вычитание. Вывод: порядок действий определен верно.

Найдем значение данного выражения.

37+9-6:2*3 =37+9-3*3=37+9-9=46-9=37

Продолжаем рассуждать.

Во втором выражении имеются скобки, значит, сначала выполняем действие в скобках, затем слева направо умножение или деление, сложение или вычитание. Проверяем: первое действие — в скобках, второе — деление, третье — сложение. Вывод: порядок действий определен неверно. Исправим ошибки, найдем значение выражения.

18:(11-5)+47=18:6+47=3+47=50

В этом выражении также имеются скобки, значит, сначала выполняем действие в скобках, затем слева направо умножение или деление, сложение или вычитание. Проверяем: первое действие — в скобках, второе — умножение, третье — вычитание. Вывод: порядок действий определен неверно. Исправим ошибки, найдем значение выражения.

7*3-(16+4)=7*3-20=21-20=1

Выполним задание.

Расставим порядок действий в выражении, используя изученное правило (рис. 5).

Рис. 5. Порядок действий

Мы не видим числовых значений, поэтому не сможем найти значение выражений, однако потренируемся применять изученное правило.

Действуем по алгоритму.

В первом выражении имеются скобки, значит, первое действие в скобках. Затем слева направо умножение и деление, потом слева направо вычитание и сложение.

Во втором выражении также имеются скобки, значит, первое действие выполняем в скобках. После этого слева направо умножение и деление, после этого — вычитание.

Проверим себя (рис. 6).

Рис. 6. Порядок действий

Сегодня на уроке мы познакомились с правилом порядка выполнения действий в выражениях без скобок и со скобками.

Список литературы

  1. М.И. Моро, М.А. Бантова и др. Математика: Учебник. 3 класс: в 2-х частях, часть 1. — М.: «Просвещение», 2012.
  2. М.И. Моро, М.А. Бантова и др. Математика: Учебник. 3 класс: в 2-х частях, часть 2. — М.: «Просвещение», 2012.
  3. М.И. Моро. Уроки математики: Методические рекомендации для учителя. 3 класс. — М.: Просвещение, 2012.
  4. Нормативно-правовой документ. Контроль и оценка результатов обучения. — М.: «Просвещение», 2011.
  5. «Школа России»: Программы для начальной школы. — М.: «Просвещение», 2011.
  6. С.И. Волкова. Математика: Проверочные работы. 3 класс. — М.: Просвещение, 2012.
  7. В.Н. Рудницкая. Тесты. — М.: «Экзамен», 2012.
  1. Festival.1september.ru ().
  2. Sosnovoborsk-soobchestva.ru ().
  3. Openclass.ru ().

Домашнее задание

1. Определи порядок действий в данных выражениях. Найди значение выражений.

2. Определи, в каком выражении такой порядок выполнения действий:

1. умножение; 2. деление;. 3. сложение; 4. вычитание; 5. сложение. Найди значение данного выражения.

3. Составь три выражения, в которых такой порядок выполнения действий:

1. умножение; 2. сложение; 3. вычитание

1. сложение; 2. вычитание; 3. сложение

1. умножение; 2. деление; 3. сложение

Найди значение этих выражений.

Тема урока: « Порядок выполнения действий в выражениях без скобок и со скобками».

Цель урока : создать условия для закрепления умений применять знания о порядке выполнения действий в выражениях без скобок и со скобками в различных ситуациях, умений решать задачи выражением.

Задачи урока.

Образовательные:

Закрепить знания учащихся о правилах выполнения действий в выражениях без скобок и со скобками; формировать у них умение пользоваться этими правилами при вычислении конкретных выражений; совершенствовать вычислительные навыки; повторить табличные случаи умножения и деления;

Развивающие:

Развивать вычислительные навыки, логическое мышление, внимание, память, познавательные способности учащихся,

коммуникативные навыки;

Воспитательные:

Воспитывать толерантное отношение друг к другу, взаимное сотрудничество,

культуру поведения на уроке, аккуратность, самостоятельность, воспитывать интерес к занятиям математикой.

Формируемые УУД:

Регулятивные УУД:

работать по предложенному плану, инструкции;

выдвигать свои гипотезы на основе учебного материала;

осуществлять самоконтроль.

Познавательные УУД:

знать правила порядка выполнения действий:

уметь разъяснить их содержание;

понимать правило порядка выполнения действий;

находить значения выражений согласно правилам порядка выполнения;

действий, используя для этого текстовые задачи;

записывать решение задачи выражением;

применять правила порядка выполнения действий;

уметь применять полученные знания при выполнении контрольной работы.

Коммуникативные УУД:

слушать и понимать речь других;

выражать свои мысли с достаточной полнотой и точностью;

допускать возможность различных точек зрения, стремиться понимать позицию собеседника;

работать в команде разного наполнения (паре, малой группе, целым классом), участвовать в обсуждениях, работая в паре;

Личностные УУД:

устанавливать связь между целью деятельности и её результатом;

определять общие для всех правила поведения;

выражать способность к самооценке на основе критерия успешности учебной деятельности.

Планируемый результат:

Предметные:

Знать правила порядка выполнения действий.

Уметь разъяснить их содержание.

Уметь решать задачи с помощью выражений.

Личностные:
Уметь проводить самооценку на основе критерия успешности учебной деятельности.

Метапредметные:

Уметь определять и формулировать цель на уроке с помощью учителя; проговаривать последовательность действий на уроке; работать по коллективно составленному плану; оценивать правильность выполнения действия на уровне адекватной ретроспективной оценки; планировать своё действие в соответствии с поставленной задачей; вносить необходимые коррективы в действие после его завершения на основе его оценки и учёта характера сделанных ошибок; высказывать своё предположение(Регулятивные УУД ).

Уметь оформлять свои мысли в устной форме; слушать и понимать речь других; совместно договариваться о правилах поведения и общения в школе и следовать им (Коммуникативные УУД ).

Уметь ориентироваться в своей системе знаний: отличать новое от уже известного с помощью учителя; добывать новые знания: находить ответы на вопросы, используя учебник, свой жизненный опыт и информацию, полученную на уроке (Познавательные УУД ).

Ход урока

1. Организационный момент.

Чтоб урок наш стал светлее,

Мы поделимся добром.

Вы ладони протяните,

В них любовь свою вложите,

И друг другу улыбнитесь.

Займите свои рабочие места.

Открыли тетради, записали число и классная работа.

2. Актуализация знаний.

На уроке нам с вами предстоит подробно рассмотреть порядок выполнения арифметических действий в выражениях без скобок и со скобками.

Устный счёт.

Игра «Найди правильный ответ».

(У каждого ученика лист с числами)

Я читаю задания, а вы, выполнив в уме действия, должны полученный результат, т. е. ответ, зачеркнуть крестиком.

    Я задумала число, из него вычла 80, получила 18. Какое число я задумала? (98)

    Я задумала число, к нему прибавила 12, получила 70. Какое число я задумала? (58)

    Первое слагаемое 90, второе слагаемое 12. Найдите сумму. (102)

Соедините полученные результаты.

Какую геометрическую фигуру вы получили? (Треугольник)

Расскажите, что вы знаете о данной геометрической фигуре. (Имеет 3 стороны, 3 вершины, 3 угла)

Продолжаем работать по карточке.

    Найдите разность чисел 100 и 22. (78)

    Уменьшаемое 99, вычитаемое 19. Найдите разность. (80).

    Возьмите число 25 4 раза. (100)

Начертите внутри треугольника еще 1 треугольник, соединяя полученные результаты.

Сколько треугольников получилось? (5)

3. Работа над темой урока. Наблюдение за изменением значения выражения от порядка выполнения арифметических действий

В жизни мы постоянно выполняем какие-либо действия: гуляем, учимся, читаем, пишем, считаем, улыбаемся, ссоримся и миримся. Эти действия мы выполняем в разном порядке. Иногда их можно поменять местами, а иногда нет. Например, собираясь утром в школу, можно сначала сделать зарядку, затем заправить постель, а можно наоборот. Но нельзя сначала уйти в школу, а потом надеть одежду.

А в математике обязательно ли выполнять арифметические действия в определенном порядке?

Давайте проверим

Сравним выражения:
8-3+4 и 8-3+4

Видим, что оба выражения совершенно одинаковы.

Выполним действия в одном выражения слева направо, а в другом справа налево. Числами можно проставить порядок выполнения действий (рис. 1).

Рис. 1. Порядок действий

В первом выражении мы сначала выполним действие вычитания, а затем к результату прибавим число 4.

Во втором выражении сначала найдем значение суммы, а потом из 8 вычтем полученный результат 7.

Видим, что значения выражений получаются разные.

Сделаем вывод: порядок выполнения арифметических действий менять нельзя .

Порядок выполнения арифметических действий в выражениях без скобок

Узнаем правило выполнения арифметических действий в выражениях без скобок.

Если в выражение без скобок входят только сложение и вычитание или только умножение и деление, то действия выполняют в том порядке, в каком они написаны.

Потренируемся.

Рассмотрим выражение

В этом выражении имеются только действия сложения и вычитания. Эти действия называют действиями первой ступени .

Выполняем действия слева направо по порядку (рис. 2).

Рис. 2. Порядок действий

Рассмотрим второе выражение

В этом выражении имеются только действия умножения и деления – это действия второй ступени.

Выполняем действия слева направо по порядку (рис. 3).

Рис. 3. Порядок действий

В каком порядке выполняются арифметические действия, если в выражении имеются не только действия сложения и вычитания, но и умножения и деления?

Если в выражение без скобок входят не только действия сложения и вычитания, но и умножения и деления, или оба этих действия, то сначала выполняют по порядку (слева направо) умножение и деление, а затем сложение и вычитание.

Рассмотрим выражение.

Рассуждаем так. В этом выражении имеются действия сложения и вычитания, умножения и деления. Действуем по правилу. Сначала выполняем по порядку (слева направо) умножение и деление, а затем сложение и вычитание. Расставим порядок действий.

Вычислим значение выражения.

18:2-2*3+12:3=9-6+4=3+4=7

Порядок выполнения арифметических действий в выражениях со скобками

В каком порядке выполняются арифметические действия, если в выражении имеются скобки?

Если в выражении имеются скобки, то сначала вычисляют значение выражений в скобках.

Рассмотрим выражение.

30 + 6 * (13 — 9)

Мы видим, что в этом выражении имеется действие в скобках, значит, это действие выполним первым, затем по порядку умножение и сложение. Расставим порядок действий.

30 + 6 * (13 — 9)

Вычислим значение выражения.

30+6*(13-9)=30+6*4=30+24=54

Правило выполнения арифметических действий в выражениях без скобок и со скобками

Как нужно рассуждать, чтобы правильно установить порядок арифметических действий в числовом выражении?

Прежде чем приступить к вычислениям, надо рассмотреть выражение (выяснить, есть ли в нём скобки, какие действия в нём имеются) и только после этого выполнять действия в следующем порядке:

1. действия, записанные в скобках;

2. умножение и деление;

3. сложение и вычитание.

Схема поможет запомнить это несложное правило (рис. 4).

Рис. 4. Порядок действий

4. Закрепление Выполнение тренировочных заданий на изученное правило

Потренируемся.

Рассмотрим выражения, установим порядок действий и выполним вычисления.

43 — (20 — 7) +15

32 + 9 * (19 — 16)

Будем действовать по правилу. В выражении 43 — (20 — 7) +15 имеются действия в скобках, а также действия сложения и вычитания. Установим порядок действий. Первым действием выполним действие в скобках, а затем по порядку слева направо вычитание и сложение.

43 — (20 — 7) +15 =43 — 13 +15 = 30 + 15 = 45

В выражении 32 + 9 * (19 — 16) имеются действия в скобках, а также действия умножения и сложения. По правилу первым выполним действие в скобках, затем умножение (число 9 умножаем на результат, полученный при вычитании) и сложение.

32 + 9 * (19 — 16) =32 + 9 * 3 = 32 + 27 = 59

В выражении 2*9-18:3 отсутствуют скобки, зато имеются действия умножения, деления и вычитания. Действуем по правилу. Сначала выполним слева направо умножение и деление, а затем от результата, полученного при умножении, вычтем результат, полученный при делении. То есть первое действие – умножение, второе – деление, третье – вычитание.

2*9-18:3=18-6=12

Узнаем, правильно ли определен порядок действий в следующих выражениях.

37 + 9 — 6: 2 * 3 =

18: (11 — 5) + 47=

7 * 3 — (16 + 4)=

Рассуждаем так.

37 + 9 — 6: 2 * 3 =

В этом выражении скобки отсутствуют, значит, сначала выполняем слева направо умножение или деление, затем сложение или вычитание. В данном выражении первое действие – деление, второе – умножение. Третье действие должно быть сложение, четвертое – вычитание. Вывод: порядок действий определен верно.

Найдем значение данного выражения.

37+9-6:2*3 =37+9-3*3=37+9-9=46-9=37

Продолжаем рассуждать.

Во втором выражении имеются скобки, значит, сначала выполняем действие в скобках, затем слева направо умножение или деление, сложение или вычитание. Проверяем: первое действие – в скобках, второе – деление, третье – сложение. Вывод: порядок действий определен неверно. Исправим ошибки, найдем значение выражения.

18:(11-5)+47=18:6+47=3+47=50

В этом выражении также имеются скобки, значит, сначала выполняем действие в скобках, затем слева направо умножение или деление, сложение или вычитание. Проверяем: первое действие – в скобках, второе – умножение, третье – вычитание. Вывод: порядок действий определен неверно. Исправим ошибки, найдем значение выражения.

7*3-(16+4)=7*3-20=21-20=1

Выполним задание.

Расставим порядок действий в выражении, используя изученное правило (рис. 5).

Рис. 5. Порядок действий

Мы не видим числовых значений, поэтому не сможем найти значение выражений, однако потренируемся применять изученное правило.

Действуем по алгоритму.

В первом выражении имеются скобки, значит, первое действие в скобках. Затем слева направо умножение и деление, потом слева направо вычитание и сложение.

Во втором выражении также имеются скобки, значит, первое действие выполняем в скобках. После этого слева направо умножение и деление, после этого – вычитание.

Проверим себя (рис. 6).

Рис. 6. Порядок действий

5. Подведение итогов.

Сегодня на уроке мы познакомились с правилом порядка выполнения действий в выражениях без скобок и со скобками. В ходе выполнения заданий определяли, зависит ли значение выражений от порядка выполнения арифметических действий, узнали, отличается ли порядок арифметических действий в выражениях без скобок и со скобками, потренировались в применении изученного правила, искали и исправляли ошибки, допущенные при определении порядка действий.

Табличка на двери

JS JavaScript Арифметические операторы



Арифметические операторы JavaScript

Арифметические операторы выполняют арифметические действия с числами (литералами или переменными).

ОператорОписание
+Дополнение
Вычитание
*Умножения
/Отдел
%Модуль (остаток)
++Увеличение
Уменьшения

Арифметические операции

Типичная арифметическая операция работает на двух числах.

Эти два числа могут быть литералами:

Пример

var x = 100 + 50;

или переменные:

Пример

var x = a + b;

или выражений:

Пример

var x = (100 + 50) * a;



Операторы и операнды

Числа (в арифметической операции) называются операндами.

Операция (выполняемая между двумя операндами) определяется оператором.

OperandОператорOperand
100+50

Добавление

Оператор сложения (+) добавляет номера:

Пример

var x = 5;
var y = 2;
var z = x + y;


Вычитание

Оператор вычитания (-) вычитает числа.

Пример

var x = 5;
var y = 2;
var z = x — y;


Умножения

Оператор умножения (*) умножает числа.

Пример

var x = 5;
var y = 2;
var z = x * y;


Разделение

Оператор деления (/) делит числа.

Пример

var x = 5;
var y = 2;
var z = x / y;


Остаток

Оператор модуля (%) возвращает остаток деления.

Пример

var x = 5;
var y = 2;
var z = x % y;

В арифметике деление двух целых чисел создает Коэффициент и остаток.
В математике результатом операции по модулю является оставшаяся часть арифметического деления.


Incrementing

The increment operator (++) increments numbers.

Пример

var x = 5;
x++;
var z = x;


Уменьшение

Оператор уменьшения (—) уменьшает число чисел.

Пример

var x = 5;
x—;
var z = x;


Приоритет оператора

Приоритет оператора описывает порядок выполнения операций в арифметическом выражении.

Пример

var x = 100 + 50 * 3;

Является результатом пример выше же, как 150 * 3, или это же, как 100 + 150?

Является ли дополнение или умножение сделано в первую очередь?

Как и в традиционной школьной математике, умножение делается в первую очередь.

Умножение (*) и деление (/) имеют более высокий приоритет , чем сложение (+) и вычитание (-).

И (как в школьной математике) приоритет может быть изменен с помощью скобок:

Пример

var x = (100 + 50) * 3;

При использовании скобок операции, находящиеся внутри скобок, рассчитываются первыми.

Когда многие операции имеют одинаковый приоритет (например, сложение и вычитание), они рассчитываются слева направо:

Пример

var x = 100 + 50 — 3;


Значения старшинства операторов JavaScript

Бледно-красные записи указывает ECMAScript 2015 (ES6) или выше.

ЗначениеОператорОписаниеПримере
20( )Группирование выражений(3 + 4)
    
19.Членperson.name
19[]Членperson[«name»]
19()Вызов функцииmyFunction()
19newСоздатьnew Date()
    
17++Приращение суффиксаi++
17Уменьшение суффиксаi—
    
16++Приращение префикса++i
16Уменьшение префикса—i
16!Логическое не!(x==y)
16typeofТипtypeof x
    
15**В возведение (ес7)10 ** 2
    
14*Умножения10 * 5
14/Division10 / 5
14%Остаток дивизиона10 % 5
    
13+Дополнение10 + 5
13Вычитание10 — 5
    
12<<Сдвиг влевоx <<2
12>>Сдвиг вправоx>> 2
12>>>Сдвиг вправо (неподписанный)x>>> 2
    
11<Менееx <y 
11<=Меньше или равноx <= y
11>Большеx> y
11>=Больше или равноx>= y
11inПравильно в объекте«PI» in Math
11instanceofЭкземпляр объектаinstanceof Array
    
10==Равноx == y
10===Строгий равныйx === y
10!=Неравныеx != y
10!==Строгие неравныеx !== y
    
9&Побитовое иx & y
8^Побитовое исключающееx ^ y
7|Побитовое илиx | y
6&&Логические иx && y
5||Логические илиx || y
4? :Состояние? «Yes» : «No»
    
3+=Назначенияx += y
3+=Назначенияx += y
3-=Назначенияx -= y
3*=Назначенияx *= y
3%=Назначенияx %= y
3<<=Назначенияx <<= y
3>>=Назначенияx>>= y
3>>>=Назначенияx>>>= y
3&=Назначенияx &= y
3^=Назначенияx ^= y
3|=Назначенияx |= y
    
2yieldФункция паузыyield x
1,Запятой5 , 6

Выражения в скобках полностью вычисляются до того, как значение используется в оставшейся части выражения.


ГБОУ школа №428 | ГБОУ школа № 428 Приморского района Санкт-Петербурга

Уважаемые родители!

В соответствии с постановлением Правительства Санкт-Петербурга от 31.01.2022 №60 «О внесении изменений в постановление Правительства Санкт-Петербурга от 13.03.2020 №121»

переводятся на дистанционное обучение ученики 7 — 11 классов
в период
со 02.02.2022 по 13.02.2022

 

Учебный год

Дата начала учебного года: 01.09.2021

Дата окончания учебного года: 31.08.2022

Дата окончания учебного периода: 25.05.2022

 

                       Начало          Окончание

I четверть     01.09.2021      24.10.2021

II четверть    04.11.2021      28.12.2021

III четверть  10.01.2022      23.03.2022

IV четверть  04.04.2022      25.05.2022

 

1 полугодие  01.09.2021      28.12.2021

2 полугодие  10.01.2022      25.05.2022

 

Каникулы   

                       Начало          Окончание

Осенние       25.10.2021      03.11.2021

Зимние         29.12.2021      09.01.2022

Весенние      24.03.2022     02.04.2022

Летние          26.05.2022     31.08.2022

 

Для обучающихся 1-х классов устанавливаются дополнительные каникулы:

с 14.02.2022 по 20.02.2022.

Расписание звонков
1 урок  9.00 – 9.45
1 перемена  9.45 – 9.55
2 урок  9.55 – 10.40
2 перемена10.40 – 11.00
3 урок11.00 – 11.45
3 перемена11.45 – 12.00
4 урок12.00 – 12.45
4 перемена12.45 – 13.05
5 урок13.05 – 13.50
5 перемена13.50 – 14.00
6 урок14.00 – 14.45
6 перемена14.45 – 14.55
7 урок14.55 – 15.40
Расписание звонков для 1-х классов 

на первое полугодие
1 урок  9.00 – 9.35
1 перемена  9.35 – 9.55
2 урок  9.55 – 10.30
Динамическая пауза      10.30 – 11.10
3 урок11.10 – 11.45
3 перемена11.45 – 12.00
4 урок12.00 – 12.35
4 перемена12.35 – 12.55
5 урок12.55 – 13.30

Родительские собрания

07.09.2021

12.10.2021

21.12.2021

15.03.2022

10.05.2022

 

Умножай и властвуй: осознанный подход без зубрежки

– Я преподаю математику в школе уже почти двадцать лет и за это время пришла к выводу, что существуют подходы, которые позволяют детям лучше проникнуть в суть предмета в сравнении с общепринятыми методиками. Таблица умножения – один из таких примеров.

Я веду одних и тех же детей с первого по одиннадцатый класс, поэтому мне хорошо видно, что в средней школе есть разница между теми учениками, кто просто вызубрил таблицу, и теми, кто осознал принципы, которые за ней стоят.

Через умножение и деление мы можем подготовить учеников к освоению понятий делимости, общих делителей и кратных, к работе с дробями, быстрому поиску общих знаменателей, сокращению дробей и так далее.

Как тетрадная обложка ввела всех в заблуждение

Когда мы говорим «таблица умножения», первое, что приходит в голову – это примеры на задней обложке тетрадей. Мы все ее помним. Но сложно сказать, почему таблица превратилась в список цифр, знание которых надо довести до автоматизма.

По моему опыту, для того чтобы дети легко осваивали умножение, детям на этот список можно не смотреть. Я предлагаю им работать с таблицей 10х10 – неважно, магнитной, сделанной из картона, написанной на доске или демонстрируемой на экране.

С моей точки зрения, именно реальная таблица – основное пособие для освоения этой темы.

Я стараюсь использовать и другие возможности, отойти от стандартного заучивания. Сейчас я работаю методистом в проекте «Яндекс.Учебник» – это сервис для учителей начальной школы со множеством заданий по математике и русскому, с автоматической проверкой.

Мы разработали стратегию обучения табличному умножению с помощью определенных заданий, что позволит детям постепенно проникнуть в суть этих операций, хорошо их освоить и закрепить.

Определение умножения

Для того чтобы умножение с помощью таблицы было естественным, я ввожу понятие умножения как количества клеток в прямоугольнике. И одновременно показываю, что традиционное определение умножения через сумму одинаковых слагаемых этому не противоречит. Ведь при пересчете клеток по строкам или по столбцам нам придется складывать одинаковые слагаемые. Я убедилась, что при геометрическом определении умножения равенство 5+5+5=3+3+3+3+3 не вызывает у детей сомнения, ведь соответствующий рисунок и подсчет клеток будут говорить сами за себя.

Начинаем работу с таблицей

Введя понятие умножения, я вешаю перед детьми готовую таблицу, и мы начинаем с ними изучать ее свойства. Я могу сказать, что дети смотрят на нее совершенно незашоренным взглядом. Они обращают внимание на симметричность относительно главной диагонали и на тот факт, что хотя клеток и сто, встречаются далеко не все числа от 1 до 100. Ученики отмечают, что некоторые числа повторяются – и насколько часто, четное ли количество раз или не обязательно, как связаны между собой числа на главной диагонали и так далее.

А затем составляем вместе с детьми свою таблицу умножения. Сто ячеек пусты или закрыты карточками. Начинаем обсуждать и подсчитывать результат, который должен оказаться в каждой ячейке. Таблица – это двумерный объект, где надо следить и за строками, и за столбцами, что само по себе очень хороший навык. Таблицей со своими учениками я пользуюсь примерно полгода. Каждый раз во время решения задач и примеров она перед глазами.

Как заполнять – двойки, пятерки, десятки и квадраты

Пока таблица перед глазами детей, я не прошу их ее заучивать. Они запоминают значения главной диагонали, то есть квадратов чисел, и доводят до автоматизма умножение на два, пять и десять.

Мой опыт говорит, что на самом деле наизусть обязательно знать всего четыре вещи. А именно: сложение двойками, пятерками, десятками и главную диагональ с квадратами чисел. Тогда остальные значения можно получить через них.

Дети, даже дошкольники, знают, что у них есть две руки, по пять пальцев на каждой, поэтому им легко считать двойками, пятерками и десятками, это дети делают задолго до того, как встречаются с идеей таблицы умножения. Таким образом, они знакомятся с понятиями умножения и деления с самого раннего возраста, просто не зная их названий.

Я прошу детей заполнить остальные ячейки таблицы, пользуясь этими основными знаниями: нужно умножить семь на шесть – это семь на пять плюс еще одна семерка. А если семь на девять, то наоборот, это семь на десять минус одна семерка.

Мы используем свойства умножения, которые пригодятся им в дальнейшем. Например, есть случаи, когда не нужно считать в столбик: мои ученики не умножают число в столбик на 199, они умножают его на 200 и вычитают одно такое число.

Если же ученик просто знает наизусть, что 6х8=48, но никогда не пробовал посчитать это как 5х8+8, то умножение останется для него пустым звуком и техническим навыком, более ничем.

Мне бывает непросто работать с детьми, которые заранее выучили таблицу умножения дома, но постепенно даже такие дети начинают видеть смысл.

Таблица умножения как фундамент для математики в средней школе

Дети постепенно осваивают операции умножения и деления и доводят их до автоматизма – осознанно, через понимание сути. Обычно к концу третьего класса мои ученики знают табличные значения наизусть, кто-то чуть раньше, кто-то немного позже. Но при этом для них становится естественным применять при вычислениях различные приемы, основанные на свойствах умножения и деления.

Например, для того чтобы число поделить на пять, его можно сначала удвоить, а потом поделить на десять. Или, например, 37 умножить на 3 – это 111, и поэтому очень легко посчитать, сколько будет 37 умножить на 24 – это же 111х8.

Пользуясь этими навыками, ученику не придется применять громоздкие вычисления в столбик, часто приводящие к ошибкам. И когда они потом получают большой пример, у них получается сделать короче, быстрее, легче. Конечно, не во всех примерах есть способ сократить – но в девяти случаях из десяти найдется.

Примеры в три действия

«В твои годы я щелкал эти задачки, как орехи», — частенько говорят детям родители, а сами впадают в ступор, безуспешно пытаясь решить простейшие примеры в три действия. Один из таких примеров мы предлагаем нашим читателям. Сложно сказать, что тому причиной, но у взрослых людей ответы на него порой получаются самыми неожиданными.

© Pexels

Примеры в три действия

Итак, предлагается решить следующий пример 12 / 2 (1 + 2). Казалось бы, о чём тут спорить? Это же математика! И чтобы получить ответ, нужно лишь правильно выстроить порядок вычислений.

© Depositphotos

Каждому школьнику известно, что арифметические действия выполняют по очереди слева направо. При этом в первую очередь делают действия в скобках, затем деление и умножение, а лишь потом сложение и вычитание. На словах просто, но на практике возникают проблемы. Иначе откуда бы появились ответы «9» и «2»?

Согласно правилам, сначала мы выполняем действия в скобках. 1 + 2 = 3
Затем слева направо:
12 / 2 = 6
6 х 3 = 18

© Depositphotos

«А вот и неправильно, — вступает в разговор поднаторевший в высшей математике папа, — знак (/) означает дробь. В таком случае всё, что после него, нужно рассматривать, как знаменатель. А значит ответ получается 2!»

© Depositphotos

Что сын-отличник должен ответить такому отцу? Я бы ответил так: «Где скобка, пап?!» В теории деление может обозначаться двоеточием (:), обелюсом (÷), косой чертой (/) или знаком дроби (—). При компьютерном вводе знак дроби для простоты часто заменяют косой чертой. Поэтому многие ошибочно воспринимают выражение «12 / 2 (1 + 2)» как дробь, где в числителе 12, а в знаменателе «2 (1 + 2)».

Однако тут очень важно помнить, что если в знаменателе у нас находится не число, а какое-то арифметическое выражение, то при записи дроби в строчку его по правилам берут в скобки. Если бы наш пример имел вид 12 / (2 (1 + 2)), то знак «/» был бы равноценен черте дроби. Но скобок за знаком деления нет, а значит воспринимать его как черту дроби в данном случае нельзя.

Отсюда 12 / 2 (1 + 2) равно 18, а не 2, как утверждают некоторые пользователи. Итак, правильный ответ — 18. А какой результат получился у тебя? Возможно, мы где-то просчитались и допустили ошибку? Выскажи свое мнение на этот счет.

Сложение и вычитание дробей. Умножение и деление дробей

Говоря о математике, нельзя не вспомнить дроби. Их изучению уделяют немало внимания и времени. Вспомните, сколько примеров вам приходилось решать, чтобы усвоить те или иные правила работы с дробями, как вы запоминали и применяли основное свойство дроби. Сколько нервов было потрачено для нахождения общего знаменателя, особенно если в примерах было больше двух слагаемых!

Давайте же вспомним, что это такое, и немного освежим в памяти основные сведения и правила работы с дробями.

Определение дробей

Начнем, пожалуй, с самого главного — определения. Дробь — это число, которое состоит из одной или более частей единицы. Дробное число записывается в виде двух чисел, разделенных горизонтальной либо же косой чертой. При этом верхнее (или первое) называется числителем, а нижнее (второе) — знаменателем.

Стоит отметить, что знаменатель показывает, на сколько частей разделена единица, а числитель — количество взятых долей или частей. Зачастую дроби, если они правильные, меньше единицы.

Теперь давайте рассмотрим свойства данных чисел и основные правила, которые используются при работе с ними. Но прежде чем мы будем разбирать такое понятие, как «основное свойство рациональной дроби», поговорим о видах дробей и их особенностях.

Какими бывают дроби

Можно выделить несколько видов таких чисел. В первую очередь это обыкновенные и десятичные. Первые представляют собой уже указанный нами вид записи с помощью горизонтальной либо косой черты. Второй вид дробей обозначается с помощью так называемой позиционной записи, когда сначала идет указание целой части числа, а затем, после запятой, указывается дробная часть.

Тут стоит отметить, что в математике одинаково используются как десятичные, так и обыкновенные дроби. Основное свойство дроби при этом действительно только для второго варианта. Кроме того, в обыкновенных дробях выделяют правильные и неправильные числа. У первых числитель всегда меньше знаменателя. Отметим также, что такая дробь меньше единицы. В неправильной дроби наоборот — числитель больше знаменателя, а сама она больше единицы. При этом из нее можно выделить целое число. В данной статье мы рассмотрим только обыкновенные дроби.

Свойства дробей

Любое явление, химическое, физическое или математическое, имеет свои характеристики и свойства. Не стали исключением и дробные числа. Они имеют одну немаловажную особенность, с помощью которой над ними можно проводить те или иные операции. Каково основное свойство дроби? Правило гласит, что если ее числитель и знаменатель умножить либо же разделить на одно и то же рациональное число, мы получим новую дробь, величина которой будет равна величине исходной. То есть, умножив две части дробного числа 3/6 на 2, мы получим новую дробь 6/12, при этом они будут равны.

Исходя из этого свойства, можно сокращать дроби, а также подбирать общие знаменатели для той или иной пары чисел.

Операции

Несмотря на то что дроби кажутся нам более сложными, по сравнению с с ними также можно выполнять основные математические операции, такие как сложение и вычитание, умножение и деление. Кроме того, есть и такое специфическое действие, как сокращение дробей. Естественно, каждое из этих действий совершается согласно определенным правилам. Знание этих законов облегчает работу с дробями, делает ее более легкой и интересной. Именно поэтому дальше мы с вами рассмотрим основные правила и алгоритм действий при работе с такими числами.

Но прежде чем говорить о таких математических операциях, как сложение и вычитание, разберем такую операцию, как приведение к общему знаменателю. Вот тут нам как раз таки и пригодится знание того, какое основное свойство дроби существует.

Общий знаменатель

Для того чтобы число привести к общему знаменателю, сначала понадобится найти наименьшее общее кратное для двух знаменателей. То есть наименьшее число, которое одновременно делится на оба знаменателя без остатка. Наиболее простой способ подобрать НОК (наименьшее общее кратное) — выписать в строчку для одного знаменателя, затем для второго и найти среди них совпадающее число. В том случае, если НОК не найдено, то есть у данных чисел нет общего кратного числа, следует перемножить их, а полученное значение считать за НОК.

Итак, мы нашли НОК, теперь следует найти дополнительный множитель. Для этого нужно поочередно разделить НОК на знаменатели дробей и записать над каждой из них полученное число. Далее следует умножить числитель и знаменатель на полученный дополнительный множитель и записать результаты в виде новой дроби. Если вы сомневаетесь в том, что полученное вами число равняется прежнему, вспомните основное свойство дроби.

Сложение

Теперь перейдем непосредственно к математическим операциям над дробными числами. Начнем с самой простой. Есть несколько вариантов сложения дробей. В первом случае оба числа имеют одинаковый знаменатель. В таком случае остается лишь сложить числители между собой. Но знаменатель не меняется. Например, 1/5 + 3/5 = 4/5.

В случае если у дробей разные знаменатели, следует привести их к общему и лишь затем выполнять сложение. Как это сделать, мы с вами разобрали чуть выше. В данной ситуации вам как раз и пригодится основное свойство дроби. Правило позволит привести числа к общему знаменателю. При этом значение никоим образом не изменится.

Как вариант, может случиться, что дробь является смешанной. Тогда следует сначала сложить между собой целые части, а затем уже дробные.

Умножение

Не требует никаких хитростей, и для того чтобы выполнить данное действие, необязательно знать основное свойство дроби. Достаточно сначала перемножить между собой числители и знаменатели. При этом произведение числителей станет новым числителем, а знаменателей — новым знаменателем. Как видите, ничего сложного.

Единственное, что от вас требуется, — знание таблицы умножения, а также внимательность. Кроме того, после получения результата следует обязательно проверить, можно ли сократить данное число или нет. О том, как сокращать дроби, мы расскажем немного позже.

Вычитание

Выполняя следует руководствоваться теми же правилами, что и при сложении. Так, в числах с одинаковым знаменателем достаточно от числителя уменьшаемого отнять числитель вычитаемого. В том случае, если у дробей разные знаменатели, следует привести их к общему и затем выполнить данную операцию. Как и в аналогичном случае со сложением, вам понадобится использовать основное свойство алгебраической дроби, а также навыки в нахождении НОК и общих делителей для дробей.

Деление

И последняя, наиболее интересная операция при работе с такими числами — деление. Она довольно простая и не вызывает особых трудностей даже у тех, кто плохо разбирается, как работать с дробями, в особенности выполнять операции сложения и вычитания. При делении действует такое правило, как умножение на обратную дробь. Основное свойство дроби, как и в случае с умножением, задействовано для данной операции не будет. Разберем подробнее.

При делении чисел делимое остается без изменений. Дробь-делитель превращается в обратную, то есть числитель со знаменателем меняются местами. После этого числа перемножаются между собой.

Сокращение

Итак, мы с вами уже разобрали определение и структуру дробей, их виды, правила операций над данными числами, выяснили основное свойство алгебраической дроби. Теперь поговорим о такой операции, как сокращение. Сокращением дроби называется процесс ее преобразования — деление числителя и знаменателя на одно и то же число. Таким образом, дробь сокращается, не меняя при этом своих свойств.

Обычно при совершении математической операции следует внимательно посмотреть на полученный в итоге результат и выяснить, возможно ли сократить полученную дробь или же нет. Помните, что в итоговый результат всегда записывается не требующее сокращения дробное число.

Другие операции

Напоследок отметим, что мы перечислили далеко не все операции над дробными числами, упомянув лишь самые известные и необходимые. Дроби также можно сравнять, преобразовать в десятичные и наоборот. Но в данной статье мы не стали рассматривать данные операции, так как в математике они осуществляются намного реже, чем те, что были приведены нами выше.

Выводы

Мы с вами поговорили о дробных числах и операциях с ними. Разобрали также основное свойство Но заметим, что все эти вопросы были рассмотрены нами вскользь. Мы привели лишь наиболее известные и употребляемые правила, дали наиболее важные, на наш взгляд, советы.

Данная статья призвана скорее освежить забытые вами сведения о дробях, нежели дать новую информацию и «забить» голову бесконечными правилами и формулами, которые, вероятнее всего, вам так и не пригодятся.

Надеемся, что материал, представленный в статье просто и лаконично, стал для вас полезным.

В математике различные типы чисел изучаются с самого своего зарождения. Существует большое количество множеств и подмножеств чисел. Среди них выделяют целые числа, рациональные, иррациональные, натуральные, четные, нечетные, комплексные и дробные. Сегодня разберем информацию о последнем множестве — дробных числах.

Определение дробей

Дроби — это числа, состоящие из целой части и долей единицы. Также, как и целых чисел, существует бесконечное множество дробных, между двумя целыми. В математике действия с дробями выполняются, так как с целыми и натуральными числами. Это довольно просто и научиться этому можно за пару занятий.

В статье представлено два вида

Обыкновенные дроби

Обыкновенные дроби представляют собой целую часть a и два числа записанных через дробную черту b/c. Обыкновенные дроби могут быть крайне удобны, если дробную часть нельзя представить в рациональном десятичном виде. Кроме того, арифметические операции удобнее производить через дробную черту. Верхняя часть называется числитель, нижняя — знаменатель.

Действия с обыкновенными дробями: примеры

Основное свойство дроби. При умножении числителя и знаменателя на одно и то же число, не являющееся нулем, в результате получается число равное данному. Это свойство дроби отлично помогает привести знаменатель для сложения (об этом будет рассказано ниже) или сократить дробь, сделать ее удобнее для счета. a/b = a*c/b*c. К примеру, 36/24 = 6/4 или 9/13 = 18/26

Приведение к общему знаменателю. Чтобы привести знаменатель дроби необходимо представить знаменатель в виде множителей, а затем помножить на недостающие числа. Например, 7/15 и 12/30; 7/5*3 и 12/5*3*2. Видим, что знаменатели отличаются двойкой, поэтому умножаем числитель и знаменатель первой дроби на 2. Получаем: 14/30 и 12/30.

Составные дроби — обыкновенные дроби с выделенной целой частью. (A b/c) Чтобы представить составную дробь в виде обыкновенной, необходимо умножить число, стоящее перед дробью на знаменатель, а затем сложить с числителем: (A*c + b)/c.

Арифметические действия с дробями

Не лишним будет рассмотреть известные арифметические действия только при работе с дробными числами.

Сложение и вычитание. Складывать и вычитать обыкновенные дроби точно так же легко, как и целые числа, за исключением одной трудности — наличия дробной черты. Складывая дроби с одинаковым знаменателем, необходимо сложить лишь числители обеих дробей, знаменатели остаются без изменения. Например: 5/7 + 1/7 = (5+1)/7 = 6/7

Если же знаменатели двух дробей представляют собой разные числа сначала нужно привести их к общему (как это сделать было рассмотрено выше). 1/8 + 3/2 = 1/2*2*2 + 3/2 = 1/8 + 3*4/2*4 = 1/8 + 12/8 = 13/8. Вычитание происходит по точно такому же принципу: 8/9 — 2/3 = 8/9 — 6/9 = 2/9.

Умножение и деление. Действия с дробями по умножению происходят по следующему принципу: отдельно перемножаются числители и знаменатели. В общем виде формула умножения выглядит так: a/b *c/d = a*c/b*d. Кроме того, по мере умножения можно сократить дробь, исключая одинаковые множители из числителя и знаменателя. Выражаясь другим языком, числитель и знаменатель делится на одно и то же число: 4/16 = 4/4*4 = 1/4.

Для деления одной обыкновенной дроби на другую, нужно поменять числитель и знаменатель делителя и выполнить умножение двух дробей, по принципу, рассмотренному ранее: 5/11: 25/11 = 5/11 * 11/25 = 5*11/11*25 = 1/5

Десятичные дроби

Десятичные дроби являются более популярной и часто используемой версией дробных чисел. Их проще записать в строчку или представить на компьютере. Структура десятичной дроби такая: сначала записывается целое число, а затем, после запятой, записывается дробная часть. По своей сути десятичные дроби — это составные обыкновенные дроби, однако их дробная часть представлена числом, деленным на кратное цифре 10. Отсюда и произошло их название. Действия с дробями десятичными аналогичны действиям с целыми числами, так как они так же записаны в десятичной системе счисления. Также в отличие от обыкновенных дробей, десятичные могут быть иррациональными. Это значит, что они могут быть бесконечны. Записываются они так 7,(3). Читается такая запись: семь целых, три десятых в периоде.

Основные действия с десятичными числами

Сложение и вычитание десятичных дробей. Выполнить действия с дробями не сложнее, чем с целыми натуральными числами. Правила абсолютно аналогичны с теми, что используют при сложении или вычитании натуральных чисел. Их точно так же можно считать столбиком, однако при необходимости заменять недостающие места нулями. Например: 5,5697 — 1,12. Для того чтобы выполнить вычитание столбиком нужно уравнять количество чисел после запятой: (5,5697 — 1,1200). Так, числовое значение не измениться и можно будет считать в столбик.

Действия с десятичными дробями нельзя производить, если одно из них имеет иррациональный вид. Для этого нужно перевести оба числа в обыкновенные дроби, а затем пользоваться приемами, описанными ранее.

Умножение и деление. Умножение десятичных дробей аналогично умножению натуральных. Их также можно умножать столбиком, просто, не обращая внимания на запятую, а затем отделить запятой в итоговом значении такое же количество знаков, сколько в сумме после запятой было в двух десятичных дробях. К примеру, 1,5 * 2,23 = 3,345. Все очень просто, и не должно вызвать затруднений, если вы уже овладели умножением натуральных чисел.

Деление также совпадает с делением натуральных чисел, но с небольшим отступлением. Чтобы разделить на десятичное число столбиком необходимо отбросить запятую в делителе, и умножить делимое на число знаков, стоявших после запятой в делителе. После чего выполнять деление как с натуральными числами. При неполном делении можно добавлять нули к делимому справа, также прибавляя ноль в ответ после запятой.

Примеры действий с десятичными дробями. Десятичные дроби — очень удобный инструмент для арифметического счета. Они сочетают в себе удобство натуральных, целых чисел и точность обыкновенных дробей. К тому же довольно просто перевести одни дроби в другие. Действия с дробями не отличаются от действий с натуральными числами.

  1. Сложение: 1,5 + 2,7 = 4,2
  2. Вычитание: 3,1 — 1,6 = 1,5
  3. Умножение: 1,7 * 2,3 = 3,91
  4. Деление: 3,6: 0,6 = 6

Кроме того, десятичные дроби подходят для представления процентов. Так, 100 % = 1; 60 % = 0,6; и наоборот: 0,659 = 65,9 %.

Вот и все, что нужно знать о дробях. В статье было рассмотрено два вида дробей — обыкновенные и десятичные. Оба довольно простые в вычислении, и если вы полностью овладели натуральными числами и действиями с ними, можете смело приступать к изучению дробных.

Действия с дробями.

Внимание!
К этой теме имеются дополнительные
материалы в Особом разделе 555.
Для тех, кто сильно «не очень…»
И для тех, кто «очень даже…»)

Итак, что из себя представляют дроби, виды дробей, преобразования — мы вспомнили. Займёмся главным вопросом.

Что можно делать с дробями? Да всё то, что и с обычными числами. Складывать, вычитать, умножать, делить.

Все эти действия с десятичными дробями ничем не отличаются от действий с целыми числами. Собственно, этим они и хороши, десятичные. Единственно, запятую правильно поставить надо.

Смешанные числа , как я уже говорил, малопригодны для большинства действий. Их всё равно надо переводить в обыкновенные дроби.

А вот действия с обыкновенными дробями похитрее будут. И гораздо важнее! Напомню: все действия с дробными выражениями с буковками, синусами, неизвестными и прочая и прочая ничем не отличаются от действий с обыкновенными дробями ! Действия с обыкновенными дробями — это основа для всей алгебры. Именно по этой причине мы очень подробно разберём здесь всю эту арифметику.

Сложение и вычитание дробей.

Сложить (отнять) дроби с одинаковыми знаменателями каждый сможет (очень надеюсь!). Ну уж совсем забывчивым напомню: при сложении (вычитании) знаменатель не меняется. Числители складываются (вычитаются) и дают числитель результата. Типа:

Короче, в общем виде:

А если знаменатели разные? Тогда, используя основное свойство дроби (вот оно и опять пригодилось!), делаем знаменатели одинаковыми! Например:

Здесь нам из дроби 2/5 пришлось сделать дробь 4/10. Исключительно с целью сделать знаменатели одинаковыми. Замечу, на всякий случай, что 2/5 и 4/10 это одна и та же дробь ! Только 2/5 нам неудобно, а 4/10 очень даже ничего.

Кстати, в этом суть решений любых заданий по математике. Когда мы из неудобного выражения делаем то же самое, но уже удобное для решения .

Ещё пример:

Ситуация аналогичная. Здесь мы из 16 делаем 48. Простым умножением на 3. Это всё понятно. Но вот нам попалось что-нибудь типа:

Как быть?! Из семёрки девятку трудно сделать! Но мы умные, мы правила знаем! Преобразуем каждую дробь так, чтобы знаменатели стали одинаковыми. Это называется «приведём к общему знаменателю»:

Во как! Откуда же я узнал про 63? Очень просто! 63 это число, которое нацело делится на 7 и 9 одновременно. Такое число всегда можно получить перемножением знаменателей. Если мы какое-то число умножили на 7, к примеру, то результат уж точно на 7 делиться будет!

Если надо сложить (вычесть) несколько дробей, нет нужды делать это попарно, по шагам. Просто надо найти знаменатель, общий для всех дробей, и привести каждую дробь к этому самому знаменателю. Например:

И какой же общий знаменатель будет? Можно, конечно, перемножить 2, 4, 8, и 16. Получим 1024. Кошмар. Проще прикинуть, что число 16 отлично делится и на 2, и на 4, и на 8. Следовательно, из этих чисел легко получить 16. Это число и будет общим знаменателем. 1/2 превратим в 8/16, 3/4 в 12/16, ну и так далее.

Кстати, если за общий знаменатель взять 1024, тоже всё получится, в конце всё посокращается. Только до этого конца не все доберутся, из-за вычислений…

Дорешайте уж пример самостоятельно. Не логарифм какой… Должно получиться 29/16.

Итак, со сложением (вычитанием) дробей ясно, надеюсь? Конечно, проще работать в сокращённом варианте, с дополнительными множителями. Но это удовольствие доступно тем, кто честно трудился в младших классах… И ничего не забыл.

А сейчас мы поделаем те же самые действия, но не с дробями, а с дробными выражениями . Здесь обнаружатся новые грабли, да…

Итак, нам надо сложить два дробных выражения:

Надо сделать знаменатели одинаковыми. Причём только с помощью умножения ! Уж так основное свойство дроби велит. Поэтому я не могу в первой дроби в знаменателе к иксу прибавить единицу. (а вот бы хорошо было!). А вот если перемножить знаменатели, глядишь, всё и срастётся! Так и записываем, черту дроби, сверху пустое место оставим, потом допишем, а снизу пишем произведение знаменателей, чтобы не забыть:

И, конечно, ничего в правой части не перемножаем, скобки не открываем! А теперь, глядя на общий знаменатель правой части, соображаем: чтобы в первой дроби получился знаменатель х(х+1), надо числитель и знаменатель этой дроби умножить на (х+1). А во второй дроби — на х. Получится вот что:

Обратите внимание! Здесь появились скобки! Это и есть те грабли, на которые многие наступают. Не скобки, конечно, а их отсутствие. Скобки появляются потому, что мы умножаем весь числитель и весь знаменатель! А не их отдельные кусочки…

В числителе правой части записываем сумму числителей, всё как в числовых дробях, затем раскрываем скобки в числителе правой части, т.е. перемножаем всё и приводим подобные. Раскрывать скобки в знаменателях, перемножать что-то не нужно! Вообще, в знаменателях (любых) всегда приятнее произведение! Получим:

Вот и получили ответ. Процесс кажется долгим и трудным, но это от практики зависит. Порешаете примеры, привыкните, всё станет просто. Те, кто освоил дроби в положенное время, все эти операции одной левой делают, на автомате!

И ещё одно замечание. Многие лихо расправляются с дробями, но зависают на примерах с целыми числами. Типа: 2 + 1/2 + 3/4= ? Куда пристегнуть двойку? Никуда не надо пристёгивать, надо из двойки дробь сделать. Это не просто, а очень просто! 2=2/1. Вот так. Любое целое число можно записать в виде дроби. В числителе — само число, в знаменателе — единица. 7 это 7/1, 3 это 3/1 и так далее. С буквами — то же самое. (а+в) = (а+в)/1, х=х/1 и т.д. А дальше работаем с этим дробями по всем правилам.

Ну, по сложению — вычитанию дробей знания освежили. Преобразования дробей из одного вида в другой — повторили. Можно и провериться. Порешаем немного?)

Вычислить:

Ответы (в беспорядке):

71/20; 3/5; 17/12; -5/4; 11/6

Умножение/деление дробей — в следующем уроке. Там же и задания на все действия с дробями.

Если Вам нравится этот сайт…

Кстати, у меня есть ещё парочка интересных сайтов для Вас.)

Можно потренироваться в решении примеров и узнать свой уровень. Тестирование с мгновенной проверкой. Учимся — с интересом!)

можно познакомиться с функциями и производными.

Примеры с дробями – один из основных элементов математики. Существует много разных типов уравнений с дробями. Ниже приведена подробная инструкция по решению примеров такого типа.

Как решать примеры с дробями – общие правила

Для решения примеров с дробями любых типов, будь то сложение, вычитание, умножение или деление, необходимо знать основные правила:

  • Для того чтобы сложить дробные выражения с одинаковым знаменателем (знаменатель – число, находящееся в нижней части дроби, числитель – в верхней), нужно сложить их числители, а знаменатель оставить тем же.
  • Для того чтобы вычесть от одного дробного выражения второе (с одинаковым знаменателем), нужно вычесть их числители, а знаменатель оставить тем же.
  • Для того чтобы сложить или вычесть дробные выражения с разными знаменателями, нужно найти наименьший общий знаменатель.
  • Для того чтобы найти дробное произведение, нужно перемножить числители и знаменатели, при этом, если есть возможность, сократить.
  • Для того чтобы разделить дробь на дробь, нужно умножить первую дробь на перевернутую вторую.

Как решать примеры с дробями – практика

Правило 1, пример 1:

Вычислить 3/4 +1/4.

Согласно правилу 1, если у дробей двух (или больше) одинаковый знаменатель, нужно просто сложить их числители. Получим: 3/4 + 1/4 = 4/4. Если у дроби числитель и знаменатель одинаковы, такая дробь будет равна 1.

Ответ: 3/4 + 1/4 = 4/4 = 1.

Правило 2, пример 1:

Вычислить: 3/4 – 1/4

Пользуясь правилом номер 2, для решения этого уравнения нужно от 3 отнять 1, а знаменатель оставить тем же. Получаем 2/4. Так как два 2 и 4 можно сократить, сокращаем и получаем 1/2.

Ответ: 3/4 – 1/4 = 2/4 = 1/2.

Правило 3, Пример 1

Вычислить: 3/4 + 1/6

Решение: Пользуясь 3-м правилом, находим наименьший общий знаменатель. Наименьшим общим знаменателем называется такое число, которое делится на знаменатели всех дробных выражений примера. Таким образом, нам нужно найти такое минимальное число, которое будет делиться и на 4, и на 6. Таким числом является 12. Записываем в качестве знаменателя 12. 12 делим на знаменатель первой дроби, получаем 3, умножаем на 3, записываем в числителе 3*3 и знак +. 12 делим на знаменатель второй дроби, получаем 2, 2 умножаем на 1, записываем в числителе 2*1. Итак, получилась новая дробь со знаменателем, равным 12 и числителем, равным 3*3+2*1=11. 11/12.

Ответ: 11/12

Правило 3, Пример 2:

Вычислить 3/4 – 1/6. Этот пример очень схож с предыдущим. Проделываем все те же действия, но в числителе вместо знака +, пишем знак минус. Получаем: 3*3-2*1/12 = 9-2/12 = 7/12.

Ответ: 7/12

Правило 4, Пример 1:

Вычислить: 3/4 * 1/4

Пользуясь четвертым правилом, умножаем знаменатель первой дроби на знаменатель второй и числитель первой дроби на числитель второй. 3*1/4*4 = 3/16.

Ответ: 3/16

Правило 4, Пример 2:

Вычислить 2/5 * 10/4.

Данную дробь можно сократить. В случае произведения сокращаются числитель первой дроби и знаменатель второй и числитель второй дроби и знаменатель первой.

2 сокращается с 4. 10 сокращается с 5. получаем 1 * 2/2 = 1*1 = 1.

Ответ: 2/5 * 10/4 = 1

Правило 5, Пример 1:

Вычислить: 3/4: 5/6

Пользуясь 5-м правилом, получим: 3/4: 5/6 = 3/4 * 6/5. Сокращаем дробь по принципу предыдущего примера и получаем 9/10.

Ответ: 9/10.


Как решать примеры с дробями – дробные уравнения

Дробными уравнениями называются примеры, где в знаменателе есть неизвестное. Для того чтобы решить такое уравнение нужно пользоваться определенными правилами.

Рассмотрим пример:

Решить уравнение 15/3x+5 = 3

Вспомним, нельзя делить на ноль, т.е. значение знаменателя не должно равняться нулю. При решении таких примеров, это нужно обязательно указывать. Для этого существует ОДЗ (область допустимых значений).

Таким образом, 3x+5 ≠ 0.
Отсюда: 3x ≠ 5.
x ≠ 5/3

При x = 5/3 уравнение просто не имеет решения.

Указав ОДЗ, наилучшим способом решить данное уравнение будет избавиться от дробей. Для это сначала представим все не дробные значения в виде дроби, в данном случае число 3. Получим: 15/(3x+5) = 3/1. Чтобы избавиться от дроби нужно умножить каждую из них на наименьший общий знаменатель. В данном случае таковым будет (3x+5)*1. Последовательность действий:

  1. Умножаем 15/(3x+5) на (3x+5)*1 = 15*(3x+5).
  2. Раскрываем скобки: 15*(3x+5) = 45x + 75.
  3. То же самое проделываем с правой частью уравнения: 3*(3x+5) = 9x + 15.
  4. Приравниваем левую и правую часть: 45x + 75 = 9x +15
  5. Переносим иксы влево, числа вправо: 36x = – 50
  6. Находим x: x = -50/36.
  7. Сокращаем: -50/36 = -25/18

Ответ: ОДЗ x ≠ 5/3 . x = -25/18.


Как решать примеры с дробями – дробные неравенства

Дробные неравенства по типу (3x-5)/(2-x)≥0 решаются при помощи числовой оси. Рассмотрим данный пример.

Последовательность действий:

  • Приравниваем числитель и знаменатель к нулю: 1. 3x-5=0 => 3x=5 => x=5/3
    2. 2-x=0 => x=2
  • Чертим числовую ось, расписывая на ней получившиеся значения.
  • Под значение рисуем кружок. Кружок бывает двух типов – заполненный и пустой. Заполненный кружок означает, что данное значение входит в ареал решений. Пустой круг говорит о том, что данное значение не входит в ареал решений.
  • Так как знаменатель не может быть равным нулю, под 2-ой будет пустой круг.


  • Чтобы определить знаки, подставляем в уравнение любое число больше двух, например 3. (3*3-5)/(2-3)= -4. значение отрицательное, значит над областью после двойки пишем минус. Затем подставляем вместо икса любое значение интервала от 5/3 до 2, например 1. Значение опять отрицательное. Пишем минус. То же самое повторяем с областью, находящейся до 5/3. Подставляем любое число, меньшее чем 5/3, например 1. Опять минус.


  • Так как нас интересуют значения икса, при котором выражение будет больше или равно 0, а таких значений нет (везде минусы), это неравенство не имеет решения, то есть x = Ø (пустое множество).

Ответ: x = Ø

Дробь — форма представления числа в математике. Дробная черта обозначает операцию деления. Числителем дроби называется делимое, а знаменателем — делитель. Например, в дроби числителем является число 5, а знаменателем — 7.

Правильной называется дробь, у которой модуль числителя больше модуля знаменателя. Если дробь является правильной, то модуль её значения всегда меньше 1. Все остальные дроби являются неправильными .

Дробь называют смешанной , если она записана как целое число и дробь. Это то же самое, что и сумма этого числа и дроби:

Основное свойство дроби

Если числитель и знаменатель дроби умножить на одно и то же число, то значение дроби не изменится, то есть, например,

Приведение дробей к общему знаменателю

Чтобы привести две дроби к общему знаменателю, нужно:

  1. Числитель первой дроби умножить на знаменатель второй
  2. Числитель второй дроби умножить на знаменатель первой
  3. Знаменатели обеих дробей заменить на их произведение

Действия с дробями

Сложение. Чтобы сложить две дроби, нужно

  1. Сложить новые числители обеих дробей, а знаменатель оставить без изменений

Пример:

Вычитание. Чтобы вычесть одну дробь из другой, нужно

  1. Привести дроби к общему знаменателю
  2. Вычесть из числителя первой дроби числитель второй, а знаменатель оставить без изменений

Пример:

Умножение. Чтобы умножить одну дробь на другую, следует перемножить их числители и знаменатели:

Деление. Чтобы разделить одну дробь на другую, следует числитель первой дроби умножить на знаменатель второй, а знаменатель первой дроби умножить на числитель второй:

Всегда ли сначала идет умножение?

Порядок операций говорит вам сначала выполнить умножение и деление , работая слева направо, прежде чем выполнять сложение и вычитание. … Далее складываем и вычитаем слева направо. (Обратите внимание, что сложение не обязательно выполняется перед вычитанием.)

Кроме того, каков правильный порядок операций?

Порядок операций — это правило, указывающее правильную последовательность шагов для вычисления математического выражения.Мы можем запомнить порядок с помощью PEMDAS: Скобки, Экспоненты, Умножение и деление (слева направо), Сложение и вычитание (слева направо) .

настоящего документа, что идет первым умножение или деление?

ПРИМЕЧАНИЕ. Несмотря на то, что Умножение стоит перед Делением в PEMDAS, они выполняются на одном шаге слева направо. Сложение и вычитание также выполняются на одном шаге.

Также, чтобы знать, что сначала умножение или сложение?
Что на первом месте в порядке операций?

  1. Умножать и делить слева направо.
  2. Складывать и вычитать слева направо.

Применяется ли Bodmas, если нет скобок?

Первоначальный ответ: Применяется ли BODMAS, когда нет скобок? Да, . Если скобок нет, следующим шагом будут индексы, затем умножение и/или деление, затем сложение и/или вычитание.

19 связанных вопросов ответы найдены


Каковы 4 порядка операций?

Порядок операций гласит, что операции должны выполняться в следующем порядке: скобки, показатели степени, умножение, деление, сложение и вычитание .

Что идет первым в порядке умножения или деления?

Экспоненты отсутствуют. Мы начинаем с умножения и деления , работая слева направо. ПРИМЕЧАНИЕ. Несмотря на то, что в PEMDAS умножение предшествует делению, они выполняются на одном шаге слева направо. Сложение и вычитание также выполняются на одном шаге.

Что на первом месте в порядке операций?

Порядок операций сообщает нам порядок решения шагов в выражениях с более чем одной операцией.Сначала решаем любые операции внутри круглых скобок или скобок . Во-вторых, мы решаем любые показатели. В-третьих, мы решаем все умножение и деление слева направо.

Каковы четыре правила математики?

Четыре правила математики: сложение, вычитание, умножение и деление .

Почему вы сначала делаете умножение?

Изучая выборку примеров в контексте, учащиеся видят, что мы умножаем, прежде чем складывать и вычитать , потому что нам нужно преобразовать группы элементов в отдельные элементы перед их объединением .Это создает основу для концептуального понимания процедур, которые они будут использовать позже, когда увидят числа вне контекста.

Что на первом месте в Бодмасе?

Правильный ответ: 43 . Правило BODMAS гласит, что мы должны сначала вычислить скобки (2 + 4 = 6), затем порядки (5 2 = 25), затем любое деление или умножение (3 x 6 (ответ в скобках) = 18), и, наконец, любое сложение или вычитание (18 + 25 = 43).

Почему Бодмас не прав?

Неправильный ответ

Его буквы обозначают Скобки, Порядок (значение сил), Деление, Умножение, Сложение, Вычитание.… Он не содержит скобок, степеней, деления или умножения, поэтому мы будем следовать BODMAS и делать сложение, за которым следует вычитание : это ошибочно.

Что такое правило DMAS?

Деление, умножение, сложение и вычитание (DMAS) — это элементарное правило для порядка выполнения двоичных операций. … DMAS, хороший инструмент, но менее убедительный/привлекательный, чтобы признать порядок его работы.

Выполняют ли калькуляторы порядок операций?

Если ваш калькулятор НЕ является научным калькулятором , он НЕ следует порядку операций и вычисляет результат в том порядке, в котором были сделаны записи.В этом случае вы не получите правильный ответ, поэтому вам придется изменить способ ввода значений.

В каком порядке вы решаете математические задачи?

Чтобы помочь учащимся в Соединенных Штатах запомнить этот порядок операций, учителя заучивают в них аббревиатуру PEMDAS: круглые скобки, показатели степени, умножение, деление, сложение, вычитание .

Почему мы делаем порядок операций?

Порядок операций — это правило, которое сообщает правильный порядок решения различных частей математической задачи .… Порядок операций важен, потому что он гарантирует, что все люди смогут читать и решать задачу одинаково.

Какое золотое правило решения уравнений?

Делайте с одной частью уравнения то же, что и с другой!

Уравнение похоже на весы. Если мы что-то надеваем или снимаем с одной стороны, шкала (или уравнение) становится несбалансированной. Решая математические уравнения, мы всегда должны поддерживать сбалансированность «шкалы» (или уравнения), чтобы обе стороны ВСЕГДА были равны .

Что такое правило DMAS?

Деление, умножение, сложение и вычитание (DMAS) — это элементарное правило для порядка выполнения двоичных операций . … DMAS, хороший инструмент, но менее убедительный/привлекательный, чтобы признать порядок его работы.

Почему вы умножаете и делите перед сложением и вычитанием?

Умножение и деление имеют такой же приоритет, как сложение и вычитание. Причина этого в том, что вычитание и деление являются вежливыми фикциями .Деление на на самом деле просто означает умножение на . Точно так же вычитание из действительно означает просто добавление к .

Что такое правило Гмдаса?

1. При особом упрощении, , если у вас есть и умножение, и деление, выполняйте операции одну за другой в порядке слева направо . … В особом упрощении, если у вас есть и сложение, и вычитание, выполняйте операции одну за другой в порядке слева направо.

Каковы золотые правила математики?

Золотое математическое правило гласит , что для любой дроби и числитель, и знаменатель можно умножить на одно и то же число без изменения значения дроби .ССЫЛКИ: Конвей, Дж. Х. и Гай, Р. К.

Всегда ли следует использовать Bodmas?

Должны ли мы все время использовать правило BODMAS, когда мы вычисляем математику? – Квора. Абсолютно не — и по двум причинам. Причина № 1: если мне нужно вычислить 2 × (3 + 4), строго следуя BODMAS, вы должны сначала выполнить сложение, потому что оно заключено в скобки, поэтому вы получите 2 × 7 = 14.

Что такое правило Бидмаса?

BIDMAS — это аббревиатура, помогающая детям запомнить порядок операций в вычислениях: Скобки, Индексы, Деление, Умножение, Сложение, Вычитание .

Кто изобрел правило Бодмаса?

Ахиллес Реселфельт — математик, который изобрел БОДМАС. Это мнемоника, которая помогает нам запомнить, как оценивать математические операторы в математическом утверждении, включающем более одной математической операции.

Это 16 или 1 математическая задача?

Некоторые люди получили 16 в качестве ответа , а некоторые люди получили 1. Путаница связана с различием между современной и исторической интерпретациями порядка операций.Правильный ответ сегодня — 16. Ответ 1 был бы правильным 100 лет назад.

Откройте для себя происхождение деления и умножения

В сегодняшней статье мы объясним происхождение математических символов деления и умножения.

Символ разделения:


Существует множество способов обозначения деления, и мы собираемся объяснить происхождение некоторых наиболее часто используемых и известных всем символов.

Горизонтальная черта дробей, введенная арабами, была впервые использована в Европе математиком Фибоначчи в тринадцатом веке, хотя ее использование не получило распространения до шестнадцатого века.

Наклонная черта, вариант горизонтальной, была введена Де Морганом в 1845 году. Это был типографский ресурс в печатных книгах, чтобы можно было писать дробь в одну строку. Символ, который сегодня широко используется для обозначения деления:
. Еще одним из знаков была скобка, хотя в настоящее время она мало используется.Чтобы выразить 21, деленное на 3, мы должны написать 21) 3 и поместить результат деления справа после еще одной скобки: 21) 3 (7.

Этот знак встречается в части Arithmetica integra (1544) немецким математиком Михаэлем Штифелем.

Этот же математик также использовал заглавные буквы M и D для обозначения умножения и деления в своей работе Deutsche Arithmetica (1545). Другие авторы также использовали букву D, в том числе как перевернутую D, например, французы Дж.Э. Галлимар (1685-1771) и другие павшие д, такие как португалец Ж. А. да Куна (1744-1787).

Один из до сих пор используемых символов деления — полоса с точками вверху и внизу. Он был введен швейцарским математиком Иоганном Генрихом Раном в его работе Teutsche Algebra (1659). Этот знак деления очень нагляден, вплоть до того, что черта дроби является общей нормой.

Этот символ не имел большого успеха в его родной стране Швейцарии или в Европе.Впрочем, так было и в Великобритании, и в США. В частности, этот символ до сих пор используется в калькуляторах для деления.

Немецкий математик Готфрид В. Лейбниц ввел две точки ( : ), и в настоящее время это наиболее широко используемый символ. Согласно Лейбницу, одно из преимуществ использования этого символа состоит в том, что деление может вестись вдоль той же линии и сохраняет связь деления с умножением, для чего Лейбниц использовал точку.

Что касается гномона или угла, который мы используем для разделения факторов деления (делимое, делитель и частное), информации не так много.

Но Бойер в своей History of Mathematics , стр. 282, говорит: «Арабы, а через них позже и европейцы, переняли большую часть своих арифметических ухищрений от индусов, и поэтому весьма вероятно, что метод «длинное деление», известное как «метод галеры» из-за его сходства с кораблем с развернутыми парусами, также происходит из Индии.Судя по всему, «метод камбуза» использовал угол, аналогичный используемому в настоящее время.

Символ умножения:

Во времена вавилонян использовали идеограмму: «а-ду». В манускрипте Бахшиили , старейшем манускрипте по индийской математике, они поставили рядом один множитель и ничего больше. Индийский математик Бхаскара Ачария (1114–1185) использовал слово «бхавита» или «бха» сразу после факторов.

Другие математики использовали букву М для умножения и букву D для деления, как мы уже говорили ранее.
В старые времена арифметики многие алгоритмы использовали крест Сан-Андрес для решения продуктов деления и умножения и пропорций. Возможно, по этой причине в 1631 году Утред выбрал этот крест как символ умножения.

Он получил широкое признание, за исключением математиков Готфрида В. Лейбница и Исаака Ньютона, которые не чувствовали себя полностью комфортно с этим символом. Лейбниц в 1698 году в одном из своих писем к математику Иоганну Бернулли пишет: «Мне не нравится символ × как символ умножения, так как его можно принять за х; … Я часто просто связываю две величины точкой, а умножение обозначаю RS · PQ.

По этой причине Лейбниц ввел точку как символ умножения.

Были и другие символы для умножения. Например, швейцарский математик Иоганн Ран (1622-1676) использовал звездочку * в своей работе Teutsche Algebra (1659). А также Лейбниц, который ранее использовал упавшую С открытой стороной вниз в своей Dissertatio комбинаторного искусства (1666).

Я надеюсь, что этот пост о делении и умножении и символах, которые мы используем для их выражения, был интересен.

Если вы хотите узнать больше о делении и умножении, зарегистрируйтесь в Smartick и попробуйте его бесплатно.

Узнать больше:

Веселье — любимый способ обучения нашего мозга

Дайан Акерман

Smartick — увлекательный способ изучения математики
  • 15 минут веселья в день
  • Адаптируется к уровню вашего ребенка
  • Миллионы учеников с 2009 года

Группа создания контента.
Мультидисциплинарная и мультикультурная команда, состоящая из математиков, учителей, профессоров и других специалистов в области образования!
Они стремятся создать наилучший математический контент.

Порядок, в котором Excel выполняет операции в формулах

В некоторых случаях порядок, в котором выполняются вычисления, может повлиять на возвращаемое значение формулы, поэтому важно понимать, как определяется порядок и как можно изменить для получения желаемых результатов.

  • Порядок расчета

    Формулы вычисляют значения в определенном порядке.Формула в Excel всегда начинается со знака равенства ( = ). Excel интерпретирует символы, следующие за знаком равенства, как формулу. После знака равенства следуют вычисляемые элементы (операнды), такие как константы или ссылки на ячейки. Они разделены операторами вычисления. Excel вычисляет формулу слева направо в соответствии с определенным порядком для каждого оператора в формуле.

  • Приоритет оператора в формулах Excel

    При объединении нескольких операторов в одну формулу Excel выполняет операции в порядке, указанном в следующей таблице.Если формула содержит операторы с одинаковым приоритетом, например, если формула содержит оператор умножения и деления, Excel вычисляет операторы слева направо.

    Оператор

    Описание

    : (двоеточие)

    (один пробел)

    , (запятая)

    Эталонные операторы

    Отрицание (как в –1)

    %

    Процент

    ^

    Возведение в степень

    * и /

    Умножение и деление

    + и –

    Сложение и вычитание

    и

    Соединяет две строки текста (объединение)

    =
    < >
    <=
    >=
    <>

    Сравнение

  • Использование скобок в формулах Excel

    Чтобы изменить порядок вычисления, заключите в круглые скобки ту часть формулы, которая будет вычисляться первой.Например, следующая формула дает 11, потому что Excel выполняет умножение перед сложением. Формула умножает 2 на 3, а затем добавляет к результату 5.

    =5+2*3

    Напротив, если вы используете круглые скобки для изменения синтаксиса, Excel складывает 5 и 2 вместе, а затем умножает результат на 3, чтобы получить 21.

    =(5+2)*3

    В следующем примере круглые скобки, заключающие первую часть формулы, заставляют Excel сначала вычислить B4+25, а затем разделить результат на сумму значений в ячейках D5, E5 и F5.

    =(B4+25)/СУММ(D5:F5)

Умножение

Умножение и Подразделение

 

 

Эти страницы обсуждают правила простого умножения и случая умножения II.

 

Простой Умножение

 

Есть для решения этой проблемы не предусмотрена дополнительная работа.Два варианта, что они сделали свою царапину работали над отдельным документом, либо выполняли арифметику в уме. В тексте не сказано, но я Интересно, определяли ли они простое умножение как умножение на меньшее число? больше или равно двенадцати или другому числу. Процесс для этой проблемы, кажется, 2 x 4 = 8, что дает нам цифру единиц. Чтобы получить цифру десятков, они выполнили (2 x 1) + (1 x 4) = 6. Этот метод продолжается до конца расчет.Для этого типа задач они не умножали 2 x 4684114 = 9368228, затем умножьте 10 x 4684114 = 46841140. Наконец, сложите два произведения 9368228. + 46841140 = 56209368.

 

Умножение Дело II

 

Теперь мы рассмотрим случай умножения II. Этот случай умножения аналогичен нашей стандартной форме. Разница лишь в том, что они не поставьте нули там, где значения не применяются.

 

 

Умножение Случай II Правило: «Множитель, помещаемый под единицы множителя под единицы десятки под десятками умножаются на каждый значащий множитель отдельно поместив первую цифру в каждом продукте точно под его множителем, затем добавьте несколько продуктов вместе в сумме в том виде, в каком они есть, и их сумма будет будет общий продукт»

 

 

 

 

 

 

 

Умножение Дело III и Дело IV

 

 

Эти страницы содержат процедуры умножения Case III и Case IV.

 

Умножение Дело III

 

Умножение Случай III имеет дело с числами, которые содержат нули справа либо от, либо от оба числа.

 

 

 

 

 

 

Мы видим что числа выстроены в линию, используя первую цифру каждого числа, которое делает не содержать нуля.Итак, проблема теперь становится 318 x 36, а нули просто переносятся вниз.

 

 

Для случае, когда оба числа содержат конечные нули, они выстраиваются в ряд, как показано на это изображение.

 

Умножение Дело IV

 

 

 

Умножение Случай IV Правило: «Когда множитель является составным числом, то есть когда он получается путем умножения любых двух чисел в таблице вместе ровно сначала на одна из этих фигур и это произведение на другое и последнее произведение будет всего требуется»

 

примеры, найденные для случая IV, похоже, не следуют правилу.По определению правила я бы подумайте, что 615243 х 144 = 88594992 становится (615243 х 12) х 12 = 88594992.

 

Умножение Случай V и деление целых чисел

 

 

 

Умножение Дело V

 

 

Умножение Случай V касается приложений и умножения.Эти проблемы умножают цену и количество, чтобы найти общую цену.

 

 

 

Далее мы см. деление целых чисел.

 

Подразделение Целые числа

 

 

формат для этих задач немного отличается от нашей стандартной формы. Разница в том, что частное помещается справа от дивиденда.

 

Дополнительный Метод деления

Схватки в делении — еще один метод. Этот метод деления сравним со случаем IV умножения.

 

 

Здесь мы есть 178464  16. Это упрощается как (178464 4) 4.

 

Конец книги по шифрованию содержит больше практических примеров.Есть также столики для одежды. мера, сухая мера, винная мера, длинная мера и твердая мера.

 

 

 

 

 

 

 

 

Как выполнять математические операции в Python 3 с операторами

Введение

Числа чрезвычайно распространены в программировании.Они используются для представления таких вещей, как размеры экрана, географические местоположения, деньги и очки, количество времени, которое проходит в видео, позиции игровых аватаров и цвета путем присвоения числовых кодов.

Умение эффективно выполнять математические операции в программировании — важный навык, который необходимо развивать, поскольку вам приходится часто работать с числами. Хотя хорошее понимание математики, безусловно, может помочь вам стать лучшим программистом, оно не является обязательным условием.Если у вас нет знаний в области математики, попробуйте думать о математике как об инструменте для достижения того, чего вы хотите достичь, и как о способе улучшить свое логическое мышление.

Мы будем работать с двумя наиболее часто используемыми числовыми типами данных Python, целыми числами и числами с плавающей запятой :

.
  • Целые числа — это целые числа, которые могут быть положительными, отрицательными или равными 0 (…, -1 , 0 , 1 , …).
  • Плавающие числа — это действительные числа, они содержат десятичную точку (как в 9.0 или -2,25 ).

В этом руководстве рассматриваются операторы, которые можно использовать с числовыми типами данных в Python.

Предпосылки

На вашем компьютере или сервере должен быть установлен Python 3 и настроена среда программирования. Если у вас не настроена среда программирования, вы можете обратиться к руководствам по установке и настройке локальной среды программирования или среды программирования на вашем сервере, подходящей для вашей операционной системы (Ubuntu, CentOS, Debian и т. д.).)

Операторы

Оператор — это символ или функция, обозначающая операцию. Например, в математике знак «плюс» или + — это оператор, обозначающий сложение.

В Python мы увидим несколько знакомых операторов, заимствованных из математики, но другие операторы, которые мы будем использовать, специфичны для компьютерного программирования.

Вот краткая справочная таблица математических операторов в Python. В этом уроке мы рассмотрим все следующие операции.

Эксплуатация Что он возвращает
х + у Сумма х и у
х — у Разница между x и у
Изменен знак x
Личность x
х * у Продукт x и y
х/у Частное x и y
х // у Частное от деления пола x и y
х % г Остаток х/у
х ** у x до y мощность

Мы также рассмотрим составные операторы присваивания, включая += и *= , которые объединяют арифметический оператор с оператором = .

Сложение и вычитание

В Python операторы сложения и вычитания работают аналогично математике. Фактически, вы можете использовать язык программирования Python в качестве калькулятора.

Информация: Чтобы следовать примеру кода в этом руководстве, откройте интерактивную оболочку Python в локальной системе, выполнив команду python3 . Затем вы можете копировать, вставлять или редактировать примеры, добавляя их после приглашения >>> .

Давайте рассмотрим несколько примеров, начиная с целых чисел:

  печать(1 ​​+ 5)
  
  [вывод вторичной_метки]
6
  

Вместо того, чтобы передавать целые числа непосредственно в оператор print , мы можем инициализировать переменные, чтобы они представляли целочисленные значения:

  а = 88
б = 103

напечатать (а + б)
  
  

Выход

191

Поскольку целые числа могут быть как положительными, так и отрицательными числами (и 0 тоже), мы можем добавить отрицательное число к положительному:

  с = -36
д = 25

печать (с + г)
  
  

Выход

-11

Дополнение будет вести себя аналогично с числами с плавающей запятой:

  е = 5.5
f = 2,5

печать (е + е)
  
  

Выход

8.0

Поскольку мы сложили вместе два числа с плавающей запятой, Python вернул значение с плавающей запятой с десятичным разрядом.

Синтаксис для вычитания такой же, как и для сложения, за исключением того, что вы измените свой оператор со знака плюс ( + ) на знак минус ( - ):

  г = 75,67
ч = 32

печать (г - ч)
  
  

Выход

43,67

Здесь мы вычли целое число из числа с плавающей запятой.Python вернет число с плавающей запятой, если хотя бы одно из чисел, участвующих в уравнении, является числом с плавающей запятой.

Унарные арифметические операции

Унарное математическое выражение состоит только из одного компонента или элемента, а в Python знаки плюс и минус могут использоваться как один элемент в паре со значением для возврата идентификатора значения ( + ) или изменения знака значения. ( - ).

Знак «плюс» используется редко, но указывает на идентичность значения.Мы можем использовать знак плюс с положительными значениями:

  я = 3,3
печать(+я)
  
  

Выход

3.3

Когда мы используем знак плюс с отрицательным значением, он также возвращает идентификатор этого значения, и в этом случае это будет отрицательное значение:

  j = -19
печать(+j)
  
  

Выход

-19

При отрицательном значении знак плюс возвращает такое же отрицательное значение.

Знак минус, наоборот, изменяет знак значения.Итак, когда мы передаем положительное значение, мы обнаружим, что знак минус перед значением возвращает отрицательное значение:

.
  я = 3,3
распечатать (-я)
  
  

Выход

-3,3

В качестве альтернативы, когда мы используем унарный оператор со знаком минус с отрицательным значением, будет возвращено положительное значение:

  j = -19
печать (-j)
  
  

Выход

19

Унарные арифметические операции, обозначенные знаком плюс и минус, вернут либо идентичность значения в случае +i , либо противоположный знак значения, как в случае -i .

Умножение и деление

Подобно сложению и вычитанию, умножение и деление будут очень похожи на то, как они делаются в математике. Знак, который мы будем использовать в Python для умножения, — * , а знак, который мы будем использовать для деления, — /.

Вот пример выполнения умножения в Python с двумя значениями с плавающей запятой:

  к = 100,1
л = 10,1

печать (к * л)
  
  

Выход

1011.0099999999999

Когда вы делите в Python 3, ваше частное всегда будет возвращаться как число с плавающей запятой, даже если вы используете два целых числа:

  м = 80
п = 5

печать (м / п)
  
  

Выход

16.0

Это одно из основных различий между Python 2 и Python 3. Подход Python 3 дает дробный ответ, поэтому при использовании / для деления 11 на 2 будет возвращено частное 5,5 . В Python 2 возвращаемое частное для выражения 11/2 равно 5 .

Оператор Python 2 / выполняет деление на этажей , где для частного x возвращаемое число является наибольшим целым числом, меньшим или равным x .Если вы запустите приведенный выше пример print(80 / 5) с Python 2 вместо Python 3, вы получите 16 в качестве вывода без десятичной точки.

В Python 3 вы можете использовать // для выполнения деления на пол. Выражение 100 // 40 вернет значение 2 . Этажное деление полезно, когда вам нужно, чтобы частное было целым числом.

Модуль

Оператор % — это модуль, который возвращает остаток, а не частное после деления.Это полезно, например, для поиска чисел, кратных одному и тому же числу.

Давайте посмотрим на модуль в действии:

  о = 85
р = 15

печать (о% р)
  
  

Выход

10

Чтобы разобрать это, 85, разделенное на 15, возвращает частное 5 с остатком 10. Значение 10 — это то, что возвращается здесь, потому что оператор по модулю возвращает остаток от выражения деления.

Если мы используем два числа с плавающей запятой по модулю, для остатка будет возвращено значение с плавающей запятой:

  q = 36.0
г = 6,0

печать (о% р)
  
  

Выход

0,0

При делении 36,0 на 6,0 остатка нет, поэтому возвращается значение 0,0 .

Мощность

Оператор ** в Python используется для возведения числа слева в степень степени справа. То есть в выражении 5 ** 3 5 возводится в третью степень. В математике мы часто видим, что это выражение представляется как 5³, а на самом деле происходит то, что 5 умножается само на себя 3 раза.В Python мы получили бы тот же результат 125 , запустив либо 5 ** 3 , либо 5 * 5 * 5 .

Давайте рассмотрим пример с переменными:

  с = 52,25
т = 7

печать (с ** т)
  
  1063173305051.292
  

Возведение числа с плавающей запятой 52,25 в степень 7 с помощью оператора ** приводит к возврату большого значения с плавающей запятой.

Приоритет оператора

В Python, как и в математике, нужно помнить, что операторы будут оцениваться в порядке старшинства, а не слева направо или справа налево.

Если мы посмотрим на следующее выражение:

  и = 10 + 10 * 5
  

Мы можем читать его слева направо, но помните, что сначала будет выполнено умножение, поэтому, если мы вызовем print(u) , мы получим следующее значение:

  

Выход

60

Это потому, что 10 * 5 оценивается как 50 , а затем мы добавляем 10 , чтобы получить 60 в качестве окончательного результата.

Если вместо этого мы хотим добавить значение 10 к 10 , а затем умножить эту сумму на 5 , мы можем использовать круглые скобки, как в математике:

  и = (10 + 10) * 5
печатать (у)
  
  

Выход

100

Один из способов запомнить порядок действий — использовать аббревиатуру PEMDAS :

.
Заказ Письмо Подставки для
1 Р P арены
2 Е E экспонент
3 М М умножение
4 Д D ivision
5 А A дополнение
6 С S вычитание

Возможно, вам знакома другая аббревиатура порядка операций, например, BEDMAS или BODMAS .Какая бы аббревиатура ни работала для вас лучше всего, старайтесь помнить о ней при выполнении математических операций в Python, чтобы возвращались ожидаемые результаты.

Операторы присваивания

Самый распространенный оператор присваивания — тот, который вы уже использовали: знак равенства = . Оператор присваивания = присваивает значение справа переменной слева. Например, v = 23 присваивает значение целого числа 23 переменной v .

При программировании обычно используются составные операторы присваивания, которые выполняют операцию над значением переменной, а затем присваивают полученное новое значение этой переменной. Эти составные операторы объединяют арифметический оператор с оператором = , поэтому для сложения мы объединим + с = , чтобы получить составной оператор += . Давайте посмотрим, как это выглядит:

  ш = 5
ш += 1
печать (ш)
  
  

Выход

6

Сначала мы установили переменную w равной значению 5 , затем мы использовали составной оператор присваивания += , чтобы добавить правильное число к значению левой переменной , а затем присвоить результат с .

Составные операторы присваивания часто используются в случае для циклов , которые вы будете использовать, когда хотите повторить процесс несколько раз:

  для x в диапазоне (0, 7):
    х *= 2
    печать (х)
  
  

Выход

0 2 4 6 8 10 12

С помощью цикла for мы смогли автоматизировать процесс оператора *= , который умножал переменную w на число 2 , а затем присваивал результат переменной w для следующей итерации цикла. для петли.

Python имеет составной оператор присваивания для каждого из арифметических операторов, обсуждаемых в этом руководстве:

  y += 1 # добавить, затем присвоить значение

y -= 1 # вычесть, затем присвоить значение

y *= 2 # умножить, затем присвоить значение

y /= 3 # разделить, затем присвоить значение

y // = 5 # разделить пол, затем присвоить значение

y **= 2 # умножить в степень затем присвоить значение

y %= 3 # вернуть остаток, затем присвоить значение
  

Составные операторы присваивания могут быть полезны, когда нужно постепенно увеличивать или уменьшать значения или когда вам нужно автоматизировать определенные процессы в вашей программе.

Заключение

В этом руководстве рассматриваются многие операторы, которые вы будете использовать с целочисленными и числовыми типами данных с плавающей запятой. Если вы хотите продолжить чтение о числах в Python, вы можете перейти к разделу Встроенные функции Python 3 для работы с числами.

Чтобы узнать больше о других типах данных, ознакомьтесь с разделом «Понимание типов данных в Python 3» и узнайте, как преобразовывать типы данных, прочитав How To Convert Data Types in Python 3.

В: Как вычислить 6/2(1+2) или 48/2(9+3)? Как обстоят дела с этим порядком работы?

Математик: Теперь, когда мы с Физик ответили на подобные вопросы по электронной почте не менее 9 раз, я полагаю, что мы должны раз и навсегда убрать эту ужасную тему с помощью короткого поста.

«Порядок операций» сообщает нам последовательность, в которой мы должны выполнять математические операции. Это всего лишь вопрос условности (не вопрос правильности или неправильности), дающий нам стандартный способ решить, является ли 6/2*3 (6/2)*3 или 6/(2*3). Конвенция была принята только для того, чтобы, когда разные люди видят уравнение, они могли согласиться с его значением, даже если они ненавидят друг друга до глубины души. Можно было бы придумать другую конвенцию, но зачем? Пусть другие люди тратят свое время на придумывание произвольных соглашений, чтобы вы могли просто тратить время на чтение сообщений о них.До нашей эры.

3. Далее умножение и деление выполняются слева направо. Итак, a*b/c*d равно ((a*b)/c)*d. Обратите внимание, что деление можно рассматривать как выполнение умножения, поскольку a/b = a*(1/b), так что в определенном смысле ни деление, ни умножение не имеют приоритета друг над другом.

4. Наконец, сложение и вычитание выполняются слева направо. Вычитание можно рассматривать как сложение, поскольку a – b = a + (-b), поэтому имеет смысл рассматривать вычитание наравне с сложением.

Люди иногда используют выражения PEMDAS, «Пожалуйста, извините, моя дорогая тетя Салли» или «Осенний снежный человек, рвотные массы, грязные куклы»,  , чтобы запомнить эти правила. Все они означают «круглые скобки, показатели степени, умножение и деление, сложение и вычитание». Эти мнемоники легко запутать, потому что они не отражают тот факт, что умножение обрабатывается так же, как деление (выполняется слева направо), а сложение обрабатывается так же, как вычитание (выполняется слева направо). ).Вот почему я предпочитаю: «Блюет выхлопной грязью, куклами, осенью и снежным человеком»

.

Что интересно отметить в порядке операций, так это то, что в них нет ничего интересного. Но это означает, что в них есть что-то интересное. Это противоречие, и, следовательно, порядок операций образует парадокс. Они также примерно так же забавны, как смотреть, как рыба ползет вверх по лестнице в качестве награды за ответы на математические вопросы.

Но это не помешает мне привести пример.3/18 + 14

— такой вопрос вам задают младшие сестры, когда они становятся достаточно взрослыми, чтобы, наконец, вроде понять, что такое математик.

Важно отметить, что не все калькуляторы обрабатывают порядок операций стандартным образом. Почему? Потому что люди, которые делают калькуляторы, ненавидят детей. Предыдущее предложение не предназначалось для утверждения фактов.

Кроме того, многие языки программирования используют стандартный порядок правил работы, но есть и исключения, такие как Smalltalk.с  = .

Эта математическая задача разделяет Интернет. Какой правильный ответ?

В настоящее время в сети бушуют величайшие дебаты нашей эпохи. Нет, дело не в том, действительно ли миру нужны постные яйца. Нет, это не недавно объявленные дебаты федеральных лидеров, частью которых является HuffPost Canada (хотя мы очень взволнованы!). И это не то, что кандидат от Демократической партии США Марианна Уильямсон придумала на дебатах во вторник вечером.

О том, как заниматься математикой.

Этот пост в Твиттере, содержащий кадр из мультфильма с математической задачей, является последней драмой, разделяющей мир.

И люди действительно разделились.

В дискуссию вступил даже местный ремонтник Queer Eye (и, вероятно, парень, который лучше всех разбирается в математике) Бобби Берк.

И Google Calculator, кажется, тоже считает, что это 16…

Но этот калькулятор считает, что это 1.

Убирайся отсюда с Google-калькулятором. Боюсь за ваше будущее 🤣🤣🤣 Любой научный калькулятор явно лучше с BIDMAS pic.twitter.com/He9MrlsesU

— specerful (@Specerful) 31 июля 2019 г.

В том-то и дело, что здесь может быть несколько правильных ответов.

я думаю, что это 1, если вы используете распределительное свойство
a(b+c) = ab+ac
в этом случае
2(2+2) = 2*2+2*2
поэтому 8/8
= 1

, но 16 из вас делают то, что в скобках, а затем выполняют операции в порядке слева направо (откуда я)
8/2 (4)
4*4
= 16

, так что, может быть, это неоднозначно? idek-

— -rae (@bigbrxthr) 31 июля 2019 г.

Это тоже, наверное, что-то региональное.Те, кто следовал BEDMAS (часто также известной как BODMAS), как правило, получали 16, а те, кто использовал PEMDAS, получали один.

BEDMAS означает скобки, экспоненты, деление, умножение, сложение и вычитание.

PEMDAS расшифровывается как Скобки, Экспоненты, Умножение, Деление, Сложение, Вычитание.

Обе аббревиатуры относятся к системам определения приоритетов математических элементов при расчете уравнения.

Быстрый опрос канадского офиса HuffPost, в основном обученного в Канаде, показывает, что мы стойкие защитники BEDMAS.Мы также в основном получили 1 в качестве нашего ответа. Принимая во внимание, что быстрый поиск в Google показывает, что PEMDAS является предпочтительным порядком операций в Соединенных Штатах.

Так какой же тогда правильный ответ? Это 16? Это 1? Это что-то совсем другое?

СМОТРЕТЬ: Математические способности лабрадора нас впечатляют.

Ну, это всегда будет двусмысленно. Многое зависит от того, как вы смотрите на 2 снаружи кронштейна. Это простое умножение, которому вы бы отдали предпочтение в порядке выбора операций? Или это дистрибутив и, следовательно, часть скобки?

Место, где вы выросли, вероятно, повлияет на ваш ответ, но математики выступают за большее количество скобок, чтобы было понятнее.

«Если это все еще кажется неясным, лучше включить [больше] квадратных скобок, чтобы устранить любую возможную двусмысленность», — сказал профессор математики британской газете Mirror. «У математиков обычно нет проблем с общением друг с другом на подобные темы, но по какой-то причине людям нравится ставить подобные задачи в социальных сетях!»

БОНУС: чехол для FOIL

Как заметил один проницательный пользователь Twitter, многие люди, возможно, забывают еще одну важную часть алгебры, которая может помочь внести ясность в эту проблему.

Умножение записывается как 8÷2(2+2), а не как 8÷2×(2+2), что позволяет предположить, что 2 является функцией скобок, а не независимым умножением.

admin

Добавить комментарий

Ваш адрес email не будет опубликован. Обязательные поля помечены *